PVS (Prashant V Shah)

– Authorized Education Provider of FPSB Ltd. (CFP Coaching and Study Material)

  • Join 808 other subscribers
  • Contact for Coaching and Study Material

    Prashant V Shah
    Ahmedabad.

    Ph: 92274 08080

    Email: pvs.cfp@gmail.com

  • Content to Purchase

    Study Texts with Pre-recorded sessions:

    Investment Planning Specialist

    Retirement and Tax Planning Specialist

    Insurance and Estate Planning

    CWM Level -2

  • Upcoming Batch

    CFP:

    Online Batch: August 2021

    Thursday: 7 pm to 9 pm Saturday: 7 pm to 9 pm
    Sunday: 11 am to 1 pm
    Fees: Rs.60,000

    Weekday Batch: July 2021

    Monday to Thursday: 4 pm to 6 pm
    Fees: Rs.75,000

    Duration: 8 months to 12 months

    CWM:

    Online Batch:

    Saturday 5 pm to 7 pm

    Sunday 9 am to 11 am

    Fees: 50,000

    Weekday Batch:

    Monday to Thursday: 2 pm to 4 pm

    Fees: 50,000

     

     

  • Blog Stats

    • 799,381 hits

Retirement Planning Practice Questions – 3, CFP

Posted by Prashant Shah on May 26, 2012

Dear Readers, Find the questions for further practice. I dont claim any ownership of questions posted below.

1

Sachin has been investing Rs. 3500 into a mutual fund at the end of each month for the last 10 years and has been earning a compound return of 12%, consisting entirely of capital appreciation. Does Sachin have enough money, after selling his investments to purchase his dream home for Rs.800000?

  • a) No because he has Rs. 563595 after sale of investment and after paying long term capital gains tax
  • b) No because he has Rs. 644108 after sale of investment and after paying long term capital gains tax
  • c) No because he has Rs. 724622 after sale of investment and after paying long term capital gains tax
  • d) Yes because he has Rs. 805135 after sale of investment
2

Sahil, age 43, can refinance Rs. 114042 at a 20-year rate for 7% and will incur closing cost of 3% of the mortgage amount to be financed in the new mortgage balance. What will be his new EMI on the mortgage under the circumstances to achieve his objective of no debt at retirement (age 60)?

  • a) Rs. 781.49
  • b) Rs. 957.56
  • c) Rs. 980.57
  • d) Rs. 986.29
3

Mira aged 30, is interested in planning for retirement. She saves Rs. 15000 per year (at the year end) in a bank fixed deposit earning 8.25% p.a. compounded annually until she retires at age 58. Her life expectancy is 80 years. What will be her corpus on the date of retirement? What is the fixed annual amount she can withdraw at the beginning of each year until age 80, in case she wishes to exhaust her corpus completely?

  • a) 1348974, 87498
  • b) 1424894, 89458
  • c) 1491655, 137767
  • d) 1491655, 91613
4

Ms. Rekha is 45 years old and plans to retire at 50. Her life expectancy is 70 years. Ms. Sushma her Financial Planner, estimates that her client will require Rs.45000 in the first month after retirement. Inflation rate is 4% p.a. and the rate of return is 6% p.a. What will be the savings per year required in order to meet this?

  • a) 1245879
  • b) 1478951
  • c) 1589420
  • d) 1689745
5

Sunil is a young professional ageing 27 years, who has started investing in a ULIP of a Insurance company. His annual contribution is Rs 60,000 in the beginning of the year. He has opted for balanced fund looking at the bear phase of the market. He is optimistic and believes that the market will rise and like wise interested in moving to growth fund, say after 2 years and is considering to a protector fund option 5 years before retirement, which as per his company.s policy is 58 years. Considering rate of return for growth fund be 12 %, balanced fund be 8 % and protector fund be 6 %, what will be the accumulated value of the ULIP if initially, investible amount of the contribution is 70 %, increasing by 10 % for subsequent years?

  • a) Rs 1,71,06,852
  • b) Rs 76,20,539
  • c) Rs 1,69,56,625
  • d) Rs 1,29,10,277
6

Ms Aditi Girpade wanted to have a retirment corpus of Rs 1.5 Crores. She has 18 years to go for retirement. But now she wants to retire 3 years earlier. What is the additional amount that she needs to invest each year if the rate is 8%?

  • a)      Rs 151921
  • b)      Rs 151912              
  • c)      Rs 151919            
  • d)      Rs 151932 
7

Akshay has a plan to save Rs 100000 each year for the next 28 years. If the amount is saved at the beginning of the year at an earning rate of 10%, what is the sum that will be accumulated by the time he retires?

  • a)      Rs 14763039        
  • b)      Rs 13420994        
  • c)      Rs 13420949        
  • d)     Rs 14763093 
8

Brijesh, age 48, plans to retire at 65 and wants to be debt free at retirement. The balance sheet mortgage is Rs. 114042 at the end of the 10th year of a 30 year loan. The monthly payment was Rs. 953.89. What was the original balance of the loan if the interest rate was 8%? (Select closest answer)

  • A) Rs. 140000
  • B) Rs. 130000
  • C) Rs. 120000
  • D) Rs. 125000
9

Gary received an inheritance of Rs. 2 Lakh. He wants to withdraw equal periodic payments at the beginning of each month for 5 years starting after 5 years. He expects to earn 12% annual interest, compounded monthly on his investments. How much can he receive each month?

  • A) Rs. 8082.28
  • B) Rs. 4448.89
  • C) Rs. 4404.84
  • D) Rs. 8002.26
10

Mr. Kalpesh decides to accumulate Rs. 50 lakh when he retires. He is 30 years old at present and wants to retire at the age of 55 years. Assume Interest Rate = 9%, Inflation = 5%. Compounding to be done on annuity certain basis. If Mr. Kalpesh could save only

Rs.45, 000 p.a. for first ten years, how much does he need to save for next 15 years to meet his retirement nest egg?

  • a)      Rs. 147008 p.a.
  • b)      Rs. 98450 p.a.
  • c)      Rs. 79250 p.a.
  • d)     Rs. 85477 p.a.
11

Ahmed’s current annual expenditure is Rs. 100000/-. He is 30 years old and expects to retire at age 55. His annual expenses are estimated to rise by 6% p.a. and his life expectancy is 75 years. His post retirement annual expenses are estimated to be 80% of his pre-retirement expenses. What will be his expenses on the first year of his retirement?

  • a)      Rs. 3,47,425/-
  • b)     Rs. 3,43,350/-
  • c)      Rs. 4,29,187/-
  • d)      Rs. 4,02,350/-
12

Ms. Rekha is 45 years old and plans to retire at 50. Her life expectancy is 70 years. Ms. Sushma, her Financial Planner, estimates that her client will require Rs.45,000/- in the first month after retirement. Inflation rate is 4% p.a. and the rate of return is 6% p.a.

What will be the savings per year required in order to meet this?

  • a)      Rs. 15,90,000/- (Approx)
  • b)      Rs. 14,80,000/- (Approx)
  • c)      Rs. 16,90,000/- (Approx)
  • d)      Rs. 12,40,000/- (Approx)
13

Avinash pays his mortgage of Rs. 12 Lakh for 15 years at an interest rate of 7%. Avinash makes the payments on a monthly basis. What is the total amount of interest Avinash will pay over the term of the mortgage? (Select closest answer)

  • a)      Rs. 6,47,000
  • b)      Rs. 7,76,300
  • c)      Rs. 7,30,200
  • d)      Rs. 7,41,480
14

Girish received an inheritance of Rs. 2 Lakh. He wants to withdraw equal periodic payments at the beginning of each month for 10 years starting after 10 years. He expects to earn 12% annual interest, compounded monthly on his investments. How much can he receive each month?

  • a)      Rs. 9,470
  • b)     Rs. 9,376
  • c)      Rs. 8,912
  • d)      Rs. 8,824
15

The cash purchase price of an item is Rs. 2,00,000. The selling company however offers installment plan, which allows an immediate payment of Rs. 10,000 and a series of 5 half-yearly payments thereafter. The first installment is payable after one and a half year. If the company wants rate of interest of 10% P.A. compounded half-yearly, what will be the half-yearly installment?

  • a)      Rs. 43,885
  • b)     Rs. 48,383
  • c)      Rs. 50,802
  • d)      Rs. 29,412
16

Sundar invests a sum of Rs. 72,000 at 5% p.a. After 7 years the rate of interest was changed to 5% p.a, compounded half yearly. After a further period of 3 years the rate was again changed to 6% p.a, compounded quarterly. What will Sundar get at the end of 15 years of commencement?

  • a)      Rs. 1,40,000
  • b)      Rs. 1,48,251
  • c)      Rs. 1,58,242
  • d)      Rs. 1,55,000
17

Alok, age 25 years, plans to retire at age 60 and his life expectancy is 75 years. His current expenditure is Rs. 2,00,000 annually. He estimates no reduction of expenses post retirement. How much will he save per annum to achieve his target, if inflation rate is 6% and expected yield from investment is 10%? Assume he wishes to leave an estate of 10% of his savings at the time of retirement.

  • a)      Rs. 73,878
  • b)      Rs. 9,612
  • c)      Rs. 66,490
  • d)      Rs. 8,651
18

Ms. Reshma is 35 years old and plans to retire at 50. Her life expectancy is 60 years. Ms. Zarina her Financial Planner, estimates that her client will require Rs.65000 in the first month after retirement. Inflation rate is 3% p.a. and the rate of return is 5% p.a. What will be the savings per year required in order to meet this?

  • a)      Rs. 328300
  • b)      Rs. 345897
  • c)      Rs. 315894
  • d)      Rs. 320458
19

Nitish is a 30 year-old self employed youth and has been using the PPF account to accumulate Rs. 30,000 per year, for his future needs. The PPF account provides a compounded return of 8% p.a. He does not have a clear view yet on financial goals and needs but has been saving as a habit, for the last 5 years. Nitish is willing to look at a lifestyle after retirement that fits into a fixed Rs. 3,00,000 per annum spend, for an estimated 15 years. What is the spending opportunity for Nitish, at the time of his retirement at 60 years, given his saving and assuming a rate of 6% on his funds after retirement? (Assumption: All computations for interest spend and savings compound annually, assuming beginning of the period investment. Answers to be rounded off to the nearest 5 rupees).

  • a)      Rs. 20,81,345
  • b)      Rs. 18,54,425
  • c)      Rs. 14,26,890
  • d)     Rs. 24,94,570
20

Ms. Mamta is 30 and plans to retire at 58 years. Her CFP says that Mamta will require inflation adjusted Rs.75000 in the first month after retirement. Inflation is 4%p.a. & return on investment is 6%p.a. What’ll be the corpus at the time of retirement in order to meet this? Will Mamta’s corpus be enough to fund her retirement if she saves Rs. 2lakhs pa. ?(end of year) . Life expectancy 75 years.

  • a)      Yes, Rs. 1,30,41,852
  • b)      Yes, Rs. 17,48,948
  • c)      Yes, Rs. 19,45,782
  • d)      No, Rs. 15,78,498

264 Responses to “Retirement Planning Practice Questions – 3, CFP”

  1. hello sir, i haven’t understood the q. no. 5 ,can you please show me the steps in short.

    • Dear Bronil,

      I will post the solution. Meanwhile request Manish to solve the question.

      Prashant.

      • rahul agarwal said

        Set = Beg
        N = 1
        I = 8
        PV=-60000*70%
        PMT=0
        FV=solve
        Py=1
        Cy=1

        Set = Beg
        N = 1
        I = 8
        PV=-Ans-60000*80%
        PMT=0
        FV=solve
        Py=1
        Cy=1

        Set = Beg
        N = 1
        I = 12
        PV=-Ans-60000*90%
        PMT=0
        FV=solve
        Py=1
        Cy=1

        Set = Beg
        N = 23
        I = 12
        PV=-Ans
        PMT=-60000
        FV=solve
        Py=1
        Cy=1

        Set = Beg
        N = 5
        I = 6
        PV=-Ans
        PMT=-60000
        FV=solve
        Py=1
        Cy=1

        Ans = 12910276.61

    • manish said

      dear bronil in order to solve the q:5 u need to understand the above mentioned 3 funds and its meaning. once u understand this clearly u can easily solve this question so lets start with the balance fund.balance fund is for the people who dont want to divesify their portfolio or dont want to invest in stocks directly, in short balance fund r ready made fund which consist of 65 % equity and the remaning 35 % consist of different debt products.2nd one is growth fund which one is invest for appreciation.3rd one is protectors fund when people transfer all or portion of their portfolio to debt instrument bcoz they are very nearer to his retirement so they dont want to take any risk in their retirement days as the safety of their pricinciple is their first priority. now lets come to our solution of the problem i solve this question with excel as through financial calc itz very confusing.so here it is:balance fund rate=8%,growth fund=12%,protector fund=6% nper=31
      age, pmt, contribution, ror, fund value
      27, 60000, 60000*70%=42000, 8%,42000*(1+8%)=45360.(here contribution is 70% of 6000 increasing by 10% for subsequent yrs)
      28, 60000,60000*80%=48000, 8%,48000+45360*(1+8%)=100828.
      29, 60000,60000*90%=54000,12%=54000+100828*(1+12%)=173408.(after 2 yrs he applied for growth fund)
      30,60000,60000*100%=60000,12%=60000+173408*(1+12%)=261417
      drag the above steps upto 52yrs of age by taking the same 12% of ror.At 52 yrs of his age is fund value is 9379403.after tht in the same way from age 53 to 57 by taking the 6 % ror u can find the answer the answer is12910276.

  2. Manish said

    Dear prashant sir my pleasure.

  3. gayatri said

    Request you to please tell me the solution of Q.4,Q.12 &Q.18

  4. manish said

    dear gayatri for question no 4 the solution are as follows:-
    as the accumulation period is 5 yrs and distribution period is 20 yrs first we need to find out how much she needs to have the corpus in order to fund her retirement days so her planner estimated tht she needs rs 45000 per month each year for 20 yrs along with that u need to calculate the real return =((1+r)/1+i)-1))=0.0192.put all these figures on excel pmt=45000,nper=240,rate=0.0192,pv=?.pv is8959710.after tht u r require to find out how much u save per year in order to achieve the corpus (i.e)8959710 in order to do this calculate the pmt.fv=8959710,nper=5,rate=6%,pmt=?.so the answer is 1589420.

    question no 12 and 18 can be solved in the same way as q no 4.

    regards,
    manish

    • gayatri said

      Thanks manish…. i scheduled my paper on next thursday & doing CFP through self study mode…if you have any tips for this paper please let me know..

    • shashikanth said

      Sir I got different answer, Q4 pmt=45000,nper=240,rate=0.0192,pv=?.pv is8959710……but here I got 2319335,

  5. manish said

    dear gayatri till now i have done with retirement planning n now planning to give insurance very soon my first experience is quite normal i haven’t face any big problem apart from one small obstacle n tht is open office as i have the thorogh practise on excel i face little bit of problem while minimizing but itz fine not a big issue,they provide u the open office instead of excel and regarding the tips i suggest u first go through the content n then do the tvm sums apart from tht i have some papers which i can provide through mail for practise n also go through the content which prshant sir has updated on this blog.

  6. gayatri said

    ok…

  7. meenakshi said

    hello sir i hv nt understood d question no.8 …can u pls help me in solving it?

    • Dear Meenakshi,

      I am requesting Manish to help you. He will reply you soon.

      Regards,
      Prashant V Shah.

    • Manish said

      Dear meenaxi u just need to find out the loan amt other details like age,retirement age is not relevant in this sum as while finding out the loan amt these details are not required these details are there just to distract u so the solution are as follow:-
      rate=8%/12
      nper=30*12
      pmt=953.89
      pv=?

      therefore the pv comes to 129999 roundabt 130000.

  8. meenakshi said

    thank u Mr. Manish and Mr. Prashant for ur help

  9. meenakshi said

    request you to please tell me the solution of ques- 13 and 17

  10. Manish said

    Dear meenaxi the solution for question no:-13 are as follow:-

    13.]the loan amt is Rs12 lakh,term of the loan is 15 yrs,rate of interest is 7%,we need to find out the interest amt of the whole term of 15 yrs.in order to find out the interest of whole term first u need to calculate the emi over the term of the loan.so emi will be calculated as follows:-

    pv=1200000
    nper=15*12
    rate=7%/12
    pmt(emi)=?.emi comes to 10785.

    emi over the term of the loan=10785*12*15=1941469.

    the difference of loan amt and emi over the term of the loan is ur total interest=1941469-1200000=741469.

    17.]in this question we need to calculate the savings per annum in order to achieve the corpus.plz note while doing this type of sums it is extremly imp to factor the inflation. the current exprenditure is Rs 2 lakh p.a so we have to find the value of this Rs 2 lakh after 35 yrs by taking the inflation rate the same are calculated as under:-

    pv=200000
    nper=35
    inflation rate=6%
    fv=?.fv comes to 1537217.

    after that u need to calculate the corpus so he can get Rs 1537217 p.a for 15 yrs at the beginning of each year.

    pmt=1537217
    nper=15
    real rate=3.77%
    type=1
    compute pv (i.e) corpus=?.so the corpus comes to 18020465.now the main trick of this sum is that this corpus represents 90% which goes into funding his retirement days.he wants to utilize the remaining 10% for leaving the estate.so we need to convert this corpus into 100% the same comes to =100*18020465/90=20022739.

    in order to achieve the above corpus he wants to know how much he should save p.a?
    fv=20022739
    nper=35
    rate=10%
    compute pmt (i.e) savings=73878.

    • meenakshi said

      Thank u so much sir …. I hv cleared my retirement module today
      Manish sir and Prashant sir u both r doing a great job ..

    • R Varadarajan said

      Dear Manish,
      In the above sum, after working out the EMI( PMT) one can go to AMRT function and choose Pmt1 and Pmt 180 & Sigma Intrerest to get the total interest.This is an alternative method and could be used to reckeck the answer.

  11. Manish said

    Dear meenaxi congratulations keep connected with this site and help others to clear the examz like u cleared today.i also wanted u to know that i am a student like u pursuing cfp so ur big thanx goes to prashant sir the creator of this blog.

    • Meenakshi said

      Thank u manish sir …and a big thanks to prashant sir …
      Sir actually i cleared all 4 modules now preparing for AFP …. Can u pls guide me how to prepare for dis exam ?????

  12. Manish said

    Dear meenakshi u can request vinayak to help u and guide you with this module as he has already cleared all the modules of cfp.

  13. Roopa Soni said

    Prashant Sir,

    Please give the solution for Q no 2.

    • Manish said

      Dear roopa the imp point to catch in this sum is to take nper-17 yrs not 20 yrs i agree that the term of the new loan is 20 yrs but he wants to debt free at age 60.therefore we take 17yrs(60-43).by considering these factors we need to calculate the emi

      refinance amt-114042
      add 3% closing cost-3421
      total loan amt-117463

      pv-117463
      nper-17*12
      rate-7%/12
      pmt-? 986.29

      • R Varadarajan said

        Dear Manish,
        With regard to the solutions for Problem no. 2, we get the answer 986.29 with ” End” mode and 880.57 with ” Begin” mode. In the case of loan repayments EMIs are normally supposed to start at the beginning. Can you please clarify why we are taking the ” End” mode in this problem. Thanks

      • R Varadarajan said

        January 25, 2013 at 11:33 am
        There was a typographical error and the message is sent again after correction.
        Dear Manish,
        With regard to the solutions for Problem no. 2, we get the answer 986.29 with ” End” mode and 980.57 with ” Begin” mode. In the case of loan repayments EMIs are normally supposed to start at the beginning. Can you please clarify why we are taking the ” End” mode in this problem. Thanks

      • Dear Varadarajan,

        Once you take the loan, the repayment always starts after a stated period (normally a month). That is the reason why we always assume end mode for the questions.

        However in real life for consumer durables loan, they have a tendancy to take 1 to 3 emi in advance. This arrangement reduces the loan taken but remember that these all are gimmicks payed to lure the customers.

        Loan gets amortized in the end mode.

        Prashant.

  14. Manish said

    Dear R varadarajan emi”s are always paid at the end of the month so no need to assume begin mode u can refer the tvm concept from this site for more clarity.

  15. R Varadarajan said

    Dear Prashant/Manish/Vinayak,
    I need your help in solving the following problem
    “Mr S aged 30 years is saving for the last 11 years in a Savings account giving a 3.5% p a. He is in the habit of increasing the amount by a fixed sum of Rs.2000 every year. If he deposits Rs.25000 in 12th Year what will be the corpus he will accumulate at the age of 60, if the rate of inflation is 11.68%
    Answers 😦 a ) 2980330 ( b ) 3251251 ( c ) 2523252 ( d )3322531
    Correct answer ( a ) 2980330”
    I will be much obliged if some one could provide a working for the same, as I am stuck with 2-3 problems of this nature.

    R Varadarajan.

  16. Altaf said

    fpsb has put notification regarding change in paper pattern . in the revised pattern are new topics are added or can we refer our old study material and refrence books prashant sir or any other members od the blog please suggest what to do thanks

    • Dear Altaf,

      I dont know which books do you use. If you have new IMS books, they are not so useful. Rest of the book are ok if you retain and update the smaller things for taxation and retirement.

      Regards,
      Prashant V Shah.

      • Altaf said

        thanks prashant sir for your help and support .sir i use old ims books published for a.y 2011-2012 as i had taken the books but was not able to give exams . are those books sufficient or do i have to consider other books .

  17. Altaf said

    thanks prashant sir for your help and support .sir i use old ims books published for a.y 2011-2012 as i had taken the books but was not able to give exams . are those books sufficient or do i have to consider other books .

    • Suma said

      Dear Prashant Shah,

      In the last two weeks I cleared 3 modules of CFP- Investment, Tax and Insurance in first attempt itself however I failed retirement and EB twice. First time I was way too distracted by a fellow exam taker at the NSE centre ( good prudence if I had just concentrated harder) and second was today. The difference between old format and new format is sky high. New format is not so scoring. First in old format 1 and 2 marks were pretty easy, so if u get even 60 marks in this section right , one had to get just 14 more marks to get it right. But in new format the questions are given as per the Syllabus listed in the notice displayed in the FPSB website. Besides the problem are much harder and time consuming. In earlier attempts any paper I could complete in one and half hour and have half hour to check. But now same is not so easy.I don’t mean to frighten anyone, but CFP exams are pretty much one of the expensive exams as against the other exams conducted at NSE centre, so try to get all concepts clear before anwering the paper. The sad part for me is my registration expires just today ( I know I pushed taking the exams way too far), and I have to re-register just to take the exam again.

      • Dear Suma,

        Congratulations for clearing 3 modules in fastrek. And consolation for rpeb.

        I too agree with you that now onwards we must be crystal clear in concepts.

        Regards,
        Prashant.

      • Altaf said

        Dear Suma

        thanks for sharing your experience i am also planning to appear for retirement planning . have you cleared the other modules in new format. wish you all the best for your preperation and prashant sir thank you for your help and support sir do us give some guidance on books to refer

      • R Varadarajan said

        Dear Suma,
        I have to share my experience in the RPEB Exams. In spite of my best preparation I could get only 86 marks( reqd 90) . although I could clear the IP and TP with high scores. As you rightly said, I could not attempt many problems for want of time since they were very confusing/time consuming. Most of the sums were far beyond the standards of the questions we had encountered either in this blog or in other work books. In fact many of the problems were even much tougher than the ones appeared in the sample paper circulated by FPSB. My confidence is completely shattered and it could take a couple of days for me to get back to normal. In case you could remember , work out/solve some of those problems, please post them here for the benefit of the others. With best wishes – R Varadarajan

  18. S said

    Dear Altaf,

    No. I was trying to clear all modules before new format sets in as was aware things are going to get only tougher. However it is just format which has changed. Only RP is something which I have taken both in old and new format. The 4 marks questions have got tougher> But if one sticks to the syllabus listed in PDF notices in FPSB links a clarity can be established.

    Good luck.

  19. R Varadarajan said

    Dear Prashantji,
    I am very disappointed to state that I could not clear the RPEB exams, today which was in the new format. Although I was well prepared, the questions were very tough and most of the problems appeared to unsolvable within the time limit specified. In fact I could not even attempt 8-10 problems which were far beyond the comprehension compared to problems we have encountered in your blog or in the question bank of IIFP. Perhaps some of them were far more difficult than the questions in the sample papers circulated by FPSB. Since I have fixed the date for the Insurance Exam next, I am planning to allow it cool a bit, as my confidence got shaken up after todays’s experience.

    I look forward to your advise.

    Thanks

    R Varadarajan

    • Altaf said

      dont be disappointed Varadarajan failure is stepping stoe towards sucess better luck next time and all the best for insurance.if possible could you please share the questions which apperead in the exam as i am planning to take the exam

      • R Varadarajan said

        Thanks Altaf. I only hope I do well in the Insurance module.
        Most of the problems were very complicated with multiple calculations with several twists in them. As indicated earlier, many of them were on the lines of the problemss in the sample questions with much more twists. There were only one or two questions which were with inflation adjusted earnings after retirement with questions for either interest or savings required. All others were with far more complicated options for investment and returns which would take perhaps more than 15 mts for each of them. Imagine far more complicated or tougher problems than the ones on the sample paper circulated by FPSB.

  20. R Varadarajan said

    Dear Altaf, I am furnishing some of the problems for your use which you could practice.
    1.A& B both aged 30 plan to save Rs.10000 pm for 30 years till their retirement. A starts immediately,invests for 10 years, stops investment and allows the investment to grow. But B starts 10 year and continues till retirement. Who gets more and how much.
    2. A&B both , both at 30 have current exp of 35000 pm and are saving till their retirement at 60, with investment return of 8.5%. The post retirement inflation and and returns are 6.5% and 7.5%. If A decides to prepone his retirement by 5 years, how soon his corpus would exhaust compared to B.
    3.Mr S & Mrs S have two annuity schemes available to them at retirement. S has expected life is 15 years while Mrs S is expected to live for 38 years. Annuity AA gives annuity of 8640 pm for S and 50% of the amount to Mrs S on his expiry. The annuity BB gives the annuity of 7700 pm for S till his survival and 90% of the amount for Mrs S. Which one is better and with how many basic point difference.
    4. There was a problem to calculate the net worth in the case of housing loan repayment and the networth calculation similar to the one on the sample paper.
    5.Find the yield of a 30 year annuity that gives 4% cashflow in the first year and an increase of 5% over the previous year for subsequent years
    6. What the Maturity date and value of a PPF accounted started on 31st March 2010 with Rs.1.00 Lakh and annual investment of Rs.1.00 Lakh on the same date every year till maturity

    You could try these and wish you all the best.

    • Suma said

      Dear Varadarajan,

      You summed up my thoughts abt RPEB. Now I dint get any slots before 1st Match in Hyd, anyways the break is long and gives me more time to prepare and reflect. Adding to what u tried let me write some questions from my memory which appeared in exam

      1. Can there be transfer between Tier I and Tier II account.
      2. There where few questions about what aspects are covered under professionalism, code of ethics.
      3. A employee is not covered under gratuity act. He has worked for 19 years, has basic last drawn salary of 105000 and 40% of basic is DA and city compensatory allowance of 30000. The basic salary was revise 3 months ago and previous to that the basic salary was 94500. He received 1250000 as gratuity. Calculate his taxable gratuity.
      4. There was one loam amt with PPF
      5. There were two problems related PPF date and withdrwal amt
      7. Max amount in EDLI
      8.There was a question in asset allocation. A person has 15 years to retirement. First 5 years he wants to allocate 2 lacs, then next 5 years 3 lacs and then 5 lacs for last 50 years in equity and debt in following ratios: first 5 years 70:30 for equity and debt, next 30:70 and again back to 70:30. The rate of interest for first 5 and last 5 years for equity is 19 and 7 and middle order block of five years is 11 and 9 for equity and debt. Calculate the retirement corpus.
      9. Salary under EPS 1995 is ___________
      10. How does the lender settle the property after reverse mortgage loan period

      Let me c i can pull from my brain later

      Suma

      • Dear Suma,

        Thank you for sharing the questions, it may help many. Many of the questions are answered in the different segments of this site.

        Did you have any question of reverse mortgage?

        Regards,
        Prashant V Shah.

      • R Varadarajan said

        Thanks Suma for sharing the questions, like I had shared the ones I remebered.I think this could help the others in the days to come. Surprisingly I didn’t get any questions on Reverse Mortgage – not even 1 mark theory question.

      • Suma said

        there is typo error it should be read as last 5 not 50 years

      • Anuj said

        Dear Prashant Sir
        what will be the solution for question no.8 (asset allocation) ?

    • Anuj said

      Dear Varadarajan what is the solution to question no. 3 (mr S and mrs. S have two annuity…) ?and in question no.2 it is not given how much they’re investing … so i am a bit confused.. ..

  21. Suma said

    yes Varadrajan, infact two

    1. An old couple has monthly expenses of 35000 and earning from fixed assets is 30000. There house was valued by a bank @35 lacs and 60% of total value was offered a RML for 15 years. If inflation is 6% and the life expectancy of last survivor of the couple is till end of tenure. Calculate the savings of old couple if they decide to invest the balance in 8% earning instrument post meeting their expenses.

    • R Varadarajan said

      Dear Suma,

      With regard the question on cash flow of 4% & 5% posted by me, I am not sure whether the question itself is complete or whether I had missed out something. W r t your question on RM problem, what is the interest rate for the loan?

    • Dear Suma,
      Solving this with some assumptions,

      – Loan is paid in lumpsum at beginning
      – Old couple is not required to repay the loan (as per the structure of RML)
      – Return on fixed assets is equal to inflation.

      PV = -35,00,000*60%
      N = 15*12
      I/Y = MNRR
      PMT = 5000 (BGN)
      FV = ? (17.40 Lakh approx)

      Pls let me know the options if you remember for modification if necessary.

      Regards,
      Prashant.

  22. Suma said

    Can someone solve this

    “5.Find the yield of a 30 year annuity that gives 4% cashflow in the first year and an increase of 5% over the previous year for subsequent years”

    Thanks

    • Vinayak said

      Dear Suma,

      You can try using the IRR function in Excel/open Office. Take the initial amount as – 100. First year amount Rs.4 and next year is 4 * (1.05) and so on. Take for 30 years. Insert the IRR function and you will get 6% as the answer, which is the yield. However I’m not very sure this is the correct approach. Let’s see what Prashant has to say about this.

      Vinayak

      • R Varadarajan said

        Dear Vinayak
        Thanks for the hint. Some how I am not getting the IRR or XIRR formula correctly even after using the formula help windows. I had taken Column 1 for years/Dates ( 1st April every year) and next column as the return as indicated But I am getting ERR report on using the IRR/XIRR format. Can you please advise. Thanks

      • Vinayak said

        Dear Varadarajan,

        You need only one column values for IRR. XIRR is when dates are also involved. In this case, since it is a regular yearly payment, you can use IRR. In one column, first enter -100 as the initial amount and in the subsequent rows, enter the yearly returns till the 30 th year. Then use the IRR function with the column values. You will get the answer. Hope its clear.
        Vinayak

      • R Varadarajan said

        Dear Vinayak,

        Thanks for the hint. I got the answer 5.93%. I trust this is correct. I recall that this question was their in my paper.I remembered the answers 6.07% , &.2, etc – but can’t recall whether this was there. Never the less, this appears to be the correct approach to the problem. Let us wait for Prashanji’s suggestion if any.

      • Suma said

        Hi Vinayak,

        Yes. IRR works out to be 6%. Has anyone tried using excel in the actual exam. I use BA II Plus financial calculator for everything else but when it comes to irr and npv prefer excel as long series of cash flow problems are easier in excel.

        Thanks once again.

        Suma

      • Vinayak said

        Dear Suma,

        Whether to use a calculator or excel in the exam depends upon the question/problem. Some problems involving corpus calcs or loan amortisation can be solved quickly using calculator. For some problems like XIRR or ULIP calculations or problems having series of cash flows, they can be solved only through excel. It really depends upon the problem and also your familiarity, speed and accuracy in using the calc and excel. Its better to be familiar with both and depending upon the problem and time, you can decide to use either calculator or excel. I personally have used only calculator ( Casio FC 200V) for all the modules except final module, in which I had to use excel for XIRR and for ULIP probs.

        Vinayak

      • Dear Vinayak,

        Its absolutely perfect.

        Prashant.

      • udaycfp said

        Dear sir
        Thank for that cash flow ans & can u help to me clear my exam

  23. Manish said

    Dear prashant sir i have some questions on reverse mortgage i m sharing here as follows:-

    1.]which of the following is not an eligibility criteria for availing loan under reverse mortgage?

    a.)indian citizen above 60 years of age.
    b.)the residential property should be free from any encumbrances.
    c.)the prospective borrower(s) should use that residential property as permanent primary residence.
    d.)the residual life of the property should be atleast 20 years
    e.)in case of married couple both should be 60 years old or more.

    2.]All payments under RML are exempt from income tax under section:

    a.)10(10D)of the income tax act,1961.
    b.)80ccc of the income tax act,1961.
    c.)10(43)of the income tax act,1961.
    d.)80ccf of the income tax act,1961.
    e.)80c of the income tax act,1961.

    3.]An individual can avail reverse mortgage loan,if he owns a

    a.)residential property
    b.)office space
    c.)land
    d.)shop
    e.)all of the above

    4.]borrower under reverse mortgage does not have the right to

    a.)repay the loan with accumulated interest and have the mortgage released without resorting to sale of the property.
    b.)use the residential property as his/her/their primary residence.
    c.)prepay the loan at any time during the loan tenor or later,without any prepayment levy.
    d.)sell the residential property on which loan is availed.

    5.]which of the following can borrow under reverse mortgage scheme?

    a.)creditworthy borrowers
    b.)senior citizens who have clear title to a house where they reside permanently.
    c.)senior citizens who have clear title to land property.
    d.)creditworthy borrowers who have clear title to land property.
    e.)creditworthy borrowers who have clear title to house property.

    6.]the maximum tenure of reverse mortgage loan can be

    a.)25 years
    b.)15 years
    c.)20 years
    d.)10 years
    e)30 years

    7.]one time reimbursement under reverse mortgage loan cannot exceed:-

    a.)500000
    b.)2500000
    c.)1500000
    d.)1000000
    e.)750000

    8.]the reverse mortgage loan shall become due and payable:-

    a.)when the first applicant dies
    b.)when the last surviving borrower dies
    c.)when the first surviving borrower dies
    d.)any of b & c
    e.)none of these

    9.]reverse mortgage loan can be availed by an individual if he is ——- years old.

    a.)58
    b.)55
    c.)60
    d.)62
    e.)65

    10.]under reverse mortgage,during the loan tenure,the borrower…

    a.)makes monthly repayments to the lender
    b.)equity grows in house
    c.)receives payments from the lender
    d.)equity declines in house
    e)c & d

    11.]monthly payments under reverse mortgage loan are capped at:-

    a.)Rs 100000
    b.)Rs 50000
    c.)Rs 200000
    d.)Rs 75000
    e.)Rs 25000

    12.]married couples will be eligible as joint borrowers under reverse mortgage loan.In such a case,the age criteria for the couple would be at the discretion of the PLI,subject to at least one of them being above 60 years of age and the other not below——-years of age.

    a.)59
    b.)52
    c.)55
    d.)50
    e.)58

    13.]In reverse mortgage loan borrower(s) or his/her heirs also:-

    a.)does not have the option of prepaying the loan
    b.)have the option of prepaying the loan
    c.)have the option of repaying the loan at the end of tenure.
    d.)does not have the option of repaying the loan
    e.)none of the above

    14.]the residual life of the property for availing reverse mortgage loan should be at least

    a.)10 years
    b.)25 years
    c.)20 years
    d.)30 years
    e.)15 years

    15.]the amount of reverse mortgage loan will depend on market value of residential property.

    a.)As assesses by the lending institution
    b.)Age of borrower(s)
    c.)interest rate
    d.)only a & b above
    e.)all of the above

    16.]under reverse mortgage loan,the PLIs would ensure that the equity of the borrower in the residential property(equity to value ratio)does not at any time during the tenor of the loan fall below

    a.)12.50%
    b.)20%
    c.)10%
    d.)15%
    e.)5%

    17.]under reverse mortgage loan,the PLIs will need to re-value the property mortgaged to them at intervals that may be fixed by the PLI depending upon the location of the property,its physical state etc.such revaluation may be done at least once every

    a.)2 years
    b.)3 years
    c.)5 years
    d.)8 years
    e)10 years

    18.]under reverse mortgage loan,the maximum lumpsum payment shall be restricted to

    a.)75% of the total eligible amount of loan subject to a cap of rs 15 lakh.
    b.)50% of the total eligible amount of loan.
    c.)75% of the total eligible amount of loan.
    d.)50% of the toal eligible amount of loan subject to a cap of rs 15 lakh
    e.)100% of the total eligible amount of loan.

    19.]loan amount under reverse mortgage loan cannot be used for

    a.)meeting any other genuine need
    b.)for supplementing pension/other income
    c.)medical,emergency expenditure for maintenance of family
    d.)up gradation,renovation and extension of residential property
    e.)speculative,trading and business purposes

    20.]the maximum period over which the payments can be made to the reverse mortgage borrower is

    a.)10 years
    b.)15 years
    c.)20 years
    d.)25 years
    e.)no restriction

    21.]under reverse mortgage,At end of loan,the borrower…

    a.)owes nothing to the lender
    b.)has substantial equity
    c.)owes substantial amount to lender
    d.)has much less,little,or no equity
    e.)c & d.

    hope the above questions help others.

  24. Altaf said

    thanks you for the questions suma,varadarajan and manish .i happened to found this set of problems with solutions which may help in additional practice please visit http://www.scribd.com/doc/48241703/Retirement-Workbook-3-2

  25. Suma said

    Dear All,

    PPF question.

    If 1 lac is deposited every year on the last working day of the FY i.e 31st March and the first deposit is made on 31st March 2013 when is the amount withdrawable.

    • R Varadarajan said

      PPF Account Opened on 31st March 2013 would mature after 31st March 2028 According to me the PPF accounts- thumb rule is 16 deposits and 15 full Financial Years.
      For example if opened on 31 March 2013 with Rs.100000 and with same deposit of Rs.100000 every year, the account will mature on 1st April 2018 with Maturity Value of Rs.3,032,428.30.
      I would like to have the views of Mr Prashant Shah.

    • Dear Suma,
      In PPF fisrt withdrawal can be made after five financial years.

      In your case
      year-1 2013-14
      year-2 2014-15
      year-3 2015-16
      year-4 2016-17
      year-5 2017-18

      hence fist withdrawal can be made on or after 1st april 2018.

  26. Suma said

    The salary considered under EPS 1995 is average salary or pensionable salary… this was one of the one mark questions… as far as I understand average salary for 12 months is treated as pensionable salary but given as a question it is confusing. Please help. And pension is determined by Pensionable salary * Pension service/ 70. Correct me if I am wrong. I know this is covered but still have a doubt whether it should be avg salary or pensionable salary as both given in options. ( sometimes even simplest things seems distant)

  27. Manish said

    Dear suma,

    the average salary here refers is named as pensionable salary in the scheme.it is arrived by considering the average salary drawn in period of 12 months preceding the date of exit membership of employee pension fund so from this explanation we can conclude that the salary considered under eps 1995 in order to compute pensionable salary is average salary and the stated formula is correct.

    • Suma said

      Yes Manish,

      But the problem is that answer had both option of pensionable salary and average salary for 12 months

      • Manish said

        Dear suma,

        According to me the answer should be average salary like i mentioned in the above comment that average salary is named as pensionable salary in the formula.For example we can put the formula like this average salary*pensionable service/70.They have just replaced the average salary with pensionable salary and therefore the actual salary considered under eps 1995 is average salary not pensionable salary.Hope its clear now.

  28. R Varadarajan said

    Dear Prashantji, Manish and Vinayak & Suma
    Today I have cleared the Insurance module and am left with the RBEB which I couldn’t clear in my previous attempt and the final module, of course. Thank you all. I look forward to the continued support and also feed back from others who have appeared for the RPEB exam recently after the modifed syllubus with type of questions they encountered. I sure Suma would have cleared RPEB by now and I look forward to her message as well.

  29. Manish said

    Dear R varadarajan congratulations and wish u all the best for rpeb.

  30. Suma said

    Dear Prashanth Sir, Vardarajan, Vinayak, Manish and other forum members,

    Looks like yesterday was a bounty. Even I cleared my RPEB module finally, now I have cleared all four modules and have to give an attempt for my AFP. Congrats Vardarajan.

    • R Varadarajan said

      Dear Suma,
      Congratulations on your success. Wish you all the best in the Finals module. Incidentally,please could you hihglight the type of questions you had received in the RPEB exams in case you remember, which will benefit us all.With best wishes,

    • Vinayak said

      Dear Suma,

      Congratulations on clearing the RPEB module and Wish you all the best for the AFP module.

      Vinayak

    • R Varadarajan said

      Dear Prashantji, Manish, Vinayak & Suma,

      I was pleasantly surprised to receive a call from FPSB saying that I had actually cleared the RPEB exam in view of the changes in the minimum requirements after the 1st February, though my results indicated otherwise. They have also updated the same in my profile on their website. I was planning to appear for the RPEB again which is not required now. Effectively, I have also cleared all the 4 papers and have to appear for the final module AFP. I now look forward to the some tips from all those who have cleared the final module and also the type of questions they have encountered.

      • Manish said

        Dear R varadarajan,

        wow thats a great news i have never heard anything like this before but now i can say that anything can happen congratulations and wish u all the very best for the final module.

      • Amazing!
        I didn’t understand what change in requirements led to change in result. Pls update all for same.

        All the best for the final.

        Regards,
        Prashant V Shah.

      • R Varadarajan said

        Dear Prashantji,
        Even I am not aware of the changes. I wrote the exam on the 14th Feb and got a score of around 58% and the result shown was that I had failed with grade’C’. But I was pleasantly surprised to receive a call from FPSB indicating that they had revised the passing criteria due to the difficulty level of the questions and since my marks were above that level I had been declared ” passed” althouhg the earlier result was otherwise. They also indicated that they would be advising all those who had appeared after 1st February and did not get through narrowly ( probably they have reduced the minimum level to 55%) about the revision in their results. I am advising this just to inform all those who had appeared for modules 1-4 after 1st April to recheck their scores and also their result in the FPSB website (OCMS Login)Perhaps some could be in for a pleasant surprise

  31. Altaf said

    Congrats Suma and Varadarajan on your sucess .suma it would be great if you can share repb questions with us

  32. vinay said

    congrats suma on clearing RPEB. I have cleared insurance module and appearing for Retirement module. Can you suggest any reference books, queston bank etc. I have study material from ACE.

  33. Anuj said

    Hi everyone,first of all congratulations to all those who have been successful. I have completed my insurance module.The same thing happened with me as Varadarajan said. I received a call from fpsb saying I have cleared RAIP. later I gave RPEB but could not clear it,probably missed it by 6 marks at the max 😦 . Well my experience has been the same,the questions are very tough and not even from the sample question from fpsb website. I have gone through the questions which some of us have shared after giving the exam which is really helpful,thanks a tonne for that. But am unable to find those kind of numericals anywhere. If anyone finds such questions do share. Apart from that I have a few doubts which are-
    1. How to calculate gratuity for piece rate employees and season employees.
    2. There is this typical problem where it is given that a person has taken a loan at floating rate of interest and later the rate changes.Now if he wants to keep the tenure same,what will be the EMI.
    3. Ms. Anita is 40 years old and wants to retire at 65. Life expectancy is 75 years. She will require 15,000 in the first month after retirement. Inflation is 4% p.a , interest rate is 7%. What is the corpus required to meet the expenses after retirement. Will the corpus be enough to fund her retirement if she saves upto Rs. 30,000 pa (at the end of the year)
    according to me the corpus comes to 15,68,545.9 but the answer given is 15,62,031. Please guide, thanks in advance.

    • vinay said

      hi, i am preparing for RPEB and tried to solve q.no- 3.–

      1) Real rate of return= ((1.07/1.04)-1)*100= 2.8846%

      2) Calculate PV- rate- 2.8846/12, nper- 120, pmt- 15000. Answer(corpus required)- 1562028.61 (nearest to your answer 1562031.)

      3) Amount saved – rate- 7%, nper- 25, pmt- 30000. FV= 1897471.13 which is enough to fund her retirement.

      Prashant sir and other experts please confirm this answer.

      • Anuj said

        the answer comes to 15,62,028 when we use set end mode and as far as I know retirement corpus is calculated using set begin. one more thing that I don’t understand is when I put i=2.8846 and P/Y=12 the answer comes to 15,68,545.9 while if put i=2.8846/12 and keep P/Y=1 the answer comes different. Shouldn’t it be the same?

      • Dear Vinay,

        There are some modifications required in this solution

        rate of return should be monthly nominal real rate (you have taken monthly effective real rate)
        withdrawal should be in beginnig mode

        Rest of the things are ok.
        This is the way to solve exams in our exams.
        Read further on https://prashantvshah.wordpress.com/2012/09/21/important-announcement-for-modifications-of-calculations-in-retirement-planning-cfp/

        Regards,
        Prashant

      • Anuj said

        I am a bit confused about the concepts of real rate,nominal rate and effective rate. from what i know real rate is the rate which is arrived after taking into account inflation,nominal rate is the one which doesn’t consider the compundings in given year and effective rate is one which does consider the no.of compoundings.
        Is my understanding right ? Also what will be the right solution to the above question ? Vinay,Prashant Sir please advice.

  34. vinay said

    Prashant sir, thank you very much for the clarification.

    Is there any short cuts in FC to calculate monthly nominal rate? In fpsb site they have used the formula – R- ( 1+r/100)^(1/n) -1.

    • Vinayak said

      Dear Vinay,
      Use the CNVR function in Financial Calculator (Casio) to find effective rate and nominal rate. For example, if you want to find monthly nominal rate given annual effective rate, go to CNVR function, put n = 12, I % = effective rate, then solve for APR, you will get the annual nominal rate. Divide this by 12 and you will get the monthly nominal rate. Once you get the concepts clear regarding effective rate and nominal rate, it’s very easy.
      Hope it’s clear.

      Vinayak

  35. vinay said

    Vinayak sir, Prashant sir

    thanks for the lucid explanation, big relief now. I use Casio FC.

  36. Deepak said

    Deepak
    hello everyone i am going to attempt RPEB on this coming monday.. I seriusly take a look on all of yours questions & solution’s and they really brings me up with lots of confidence.. thanks a lot. u all r really doing a great job.. !

  37. Suma said

    Hello everyone,

    I am giving my first shot at AFP tommorrow. I dare say I prepared, entire time flew by for end of FY work. I am only hoping to just go sit through and ateempt as many as I know. Request reply for confirming few interest rates as rates have changed on 1st April 2013 ( dont know if I have to go by old rates)

    Instrument old rate New rate

    PPF 8.8 8.7
    NSC 5 8.6 8.5
    NSC10 8.9 8.8

    Please confirm

    • Dear Suma,

      The applicable rates for our exams are the rates which were declared in April,2012 and not April 2013. Hence for PPF you can go ahead with 8.8% as an applicable rate

      • R Varadarajan said

        Dear Prashantji, Vinayak, Manish, Suma,
        I am pleased to advise that I have cleared the Fianal Module today. Although the grade is not great, I am happy I could clear the same since most of the 5 marks questions were very tough and confusing with mutiple investment vehicles/options with different periods for switching. I would request all the members to prepare very thoroghly with regard to TVM for periods half yearly and quarterly invesment options besides monthly and annual options

      • Dear Varadarajan,

        Heartily congratulations..

        The initiative which we all took has started paying. Remain associated and keep helping others.

        Wish all the luck for the future.

        Regards,
        Prashant V Shah.

      • R Varadarajan said

        Thanks Prashantji for your guidance all along and for your good wishes. Yes. I will actively be invloved with the group and will be only too glad to be of some use to others. Thank you once again.

      • Vinayak said

        Mr. Varadarajan,

        Congratulations on clearing the final module. It really is very commendable.

        Vinayak

      • R Varadarajan said

        Dear Vinaya,
        I forgot the most important thing !!!!. To Thank Prashantji, Manish, Vinayak and Suma for the timely support through thsi blog. A BIG THANK YOU !!!!

      • Manish said

        Dear R Varadarajan,

        well done and congratulations for achieving the success in finale. Hope to see you in future for further interaction through this blog.Meanwhile All the best for your future.

    • R Varadarajan said

      Dear Suma,

      I am sure you had the opportunity to see Prashantji’s clarifications on interest. WISH YOU BEST OF LUCK IN THE FINAL EXAM

    • burranaresh said

      can you plz provide me workbook/practice questions for retirement module as i am preparing in self study mode..

      thank you in advance..

  38. ramakrishna said

    Hi,
    I am Ramkrishna, I cleared Risk and insurance module on 16th March and I took retirement module on 28th march and 16th April both the times I got only D grade.
    Can any one suggest some good books to prepare retirement modle. I had material from ROOTS.

    Dear Suma,Vardharajan,

    Please help me to clear the retirement module.

    Thanks & Regards,
    Ramakrishna

    • Dear Ramakrishna,

      Roots material must be fine. You can refer to the RPEB segment for practice question. We all are here to help you for further queries.

      Regards,
      Prashant V Shah.

    • burranaresh said

      can you plz provide me workbook/practice questions for retirement module as i am preparing in self study mode..

      thank you in advance..

  39. ramakrishna said

    Hi Prashant,

    Thank youvery much for your kind reply.

    But the problem with new patteren exam is very time consuming & difficulty level increased.

    I attempted All the 68 marks of 2 & one marks questions atleast i will get minimum 50 marks out 68.

    but I attempted only 11 questions of 3 marks.
    4 marks question I am not touched because of time constraint.

    The old material question almost very simple and direct now in exam each question is bit twisting.

    For example.

    A old couple having monthly expenses of 35000 PM and they are worried about infilation 6% growing year by year.
    Couple is earning 30000 PM from fixed assets and bank offered a RML of 36 lakhs for 15 years of interest rate of 9%
    P.A life expectency of an last surviour is end of loan tenure.calculate thesavings ofold coupleif they invest balance amountwith 8% earning savings instrument.

    Solution Which I did in Exam
    The balance expenses of old couple = Monthly exp – Fixed earning income.

    35000-30000 = 5000

    RML part

    PV= 3600000
    I= 9/12
    N= 15*12

    EMI = 35499.55 mode is END

    Monthly expense will grow 6% P.A
    Difference amount available for saving monthly = 35499.33- 5000
    30499.33

    which will grow as a infilation adjusted rate of raturn.
    ROR monthly coverted is =(((1.08/1.060)-1)*1000/12
    0.155889156 after convert function using in FC200V
    N= 180
    I =0.155889156
    pmt=39499.33
    FV= 6331845.795 (as saving of old couple)

    Another question

    An old couple receiving Rs 26400 monthly pension , this amount is sufficient for their monthly expense and a bank is offering 60% of 36L as a RML with a 9% rate of interest P.A for 15 years. what is the additional income of the couple monthly/

    Solution

    PV = 60*3600000/100 = 2160000
    N= 12*15 = 180
    I = 9/12 = .75
    Pmt = 21908.158 (mode is end)

    please verify my solution and correct me if did any wrong in the above solutions.

    Thanks & Regards,
    Ramakrishna Rao

    • Manish said

      Dear Ramakrishna,

      Request you to see the comment number 21 in order to know the solution of first question.The solution of second question is:-

      fv=3600000*60%=2160000
      nper=15*12=180
      i=9/12=0.75
      pmt=?(5708).

      in reverse mortgage loan amount is always fv.

      • ramakrishna said

        Dear Manish,

        Thank you very much for you kind suggestionI took RML Value as a PV in both the problems you mean to say always in the RML problems value is FV.

        Thanks & Regards,
        Ramakrishna Rao

      • akash singhal said

        shouldn’t the type be 1 in this solution as the amount is required at the beggining of the month always?

      • Many solutions might have been as per old course.
        Current course suggest that investment normally happens at the beginning of the period.

        Regards,
        prashant.

  40. ramakrishna said

    FPSB board itself selling some material 4 work books + 4 concepts books will cost I think 3500
    I think they updated with new patteren of exam.
    Please provide me your suggestion on this

    • Anuj said

      Dear Ramakrishna, are you talking about these ?

      Introduction to Financial Planning – Taxmann Publications/IIBF
      Insurance Products (Including Pension Products) – Taxmann Publications/IIBF
      IC-33 Life Insurance – Insurance Institute of India
      IC-34 General Insurance – Insurance Institute of India
      Mutual Fund Products & Services – Taxmann Publications/IIBF
      In the wonderland of Investment – A.N. Shanbag (Standard Book Co.)
      Students Guide To Income tax (Student Edition) – Dr. Vinod k. Singhania (Taxmann Pub.)
      Investment Analysis & Portfolio Management – Dr. Prasanna Chandra (McGraw – Hill)

      These are for 3000 Rs. If these aren’t the one you referring to, please provide a link to their names…

      • ramakrishna said

        Hi Anju,
        For Insurance Planning I read IC 33 & IC34 material and IMS pro school material so That I cleared the RAIP paper.

        Now I am Planning to take Investment planning I will by Prassana chandra now,

        Regards,
        Ramakrishna

  41. vinay said

    sir
    find below the sample question and answer from fpsb site.
    my doubt is how to solve this probem if the rate of increase of rent and rate of increase of expenses are different? ie say expenses increase by 8%pa and rent by 7% pa.
    i still did not understand the last step of calculation ie number of years the corpus would last and the logic of accommodating the growing expense, growing rent and the last- rate of return.
    please help.

    Section II- question 6–
    A 45-year old man spends Rs. 7.5 lakh p.a., almost the amount he earns, to maintain his family. He expects his expenses to rise by 7% p.a. He has not saved for retirement. He has a second house which he wants to rent at Rs. 20,000 p.m. immediately, the rent expected to increase by 7 % p.a. You advise him to create a corpus by his age of 60 by investing the rent received in an instrument yielding 9% p.a. at the end of every year. You estimate the number of years the accumulated corpus would last taking the received rents post-retirement into account. The same is ____.
    Solution:
    Current expenses 750,000Rs. p.a.
    Rate of increase of expenses 7% p.a.
    Rent received from renting second house 240,000 Rs. p.a.
    Rate of increase of rent 7%p.a.
    Investment yield from investing rent 9%p.a.
    Accumulated value of investment at age 60- 10,601,411Rs .- 240000*((1+9%)^15-(1+7%)^15)/(9%-7%)
    Expenses required at age 60-2,069,274 Rs. p.a. 750000*(1+7%)^15
    Expenses covered by rent received at age 60—662,168Rs. p.a. 240000*(1+7%)^15
    Balance expenses to be drawn from corpus 1,407,106Rs. p.a. — 2069274-662168
    Number of years the corpus would last – 8 NPER((1+9%)/(1+7%)-1,-1407106,10601411,0,1)
    Answer- 8.

  42. Manish said

    Dear vinay,

    In that case as stated by you, more expenses will be required at age 60 while the rent rises at the same rate as mentioned in standard question.As expenses rises more than the rental income the accumulated corpus will vanish very early.

    ‘The number of years the corpus would last’ means how many years your accumulated corpus would support you.In your case it should be less because your expenses rises more than the rental income you can find the same in calculation as under :-

    1.)Find the accumulated corpus by taking the rate of return as 9% and also increase your investment every year by 7% as your rent rises by 7% .you can do this calculation by using the growing annuity formula or you can do this manually on excel either way you are familiar with.

    accumulated corpus comes to 10601411.

    2.)Find the expenses required at age 60.

    pv =750000
    rate=8%
    nper=15
    fv=?2379126.

    3.)Find the expected rent every year after 15 yrs.

    pv=240000
    rate=7%
    nper=15
    fv?662167.

    4.)now from the retirement age every year you need 2379126 in order to maintain the same life style.

    therefore expenses required = 2379126
    less:-expenses covered by rent=662167

    balance expenses to be drawn from corpus=1716959

    5.)How long your corpus would last?

    pmt=1716959
    real rate=0.92%
    pv= -10601411
    nper=?6 yrs.

    Now you can compare this with the main question.

    • vinay said

      manish sir,
      thanks for the reply. my doubt here is — is it assumed that expenses will not grow post- retirement? i presume that the real rate in the last step accommodates both rate of interest and the rate of growth of rent. if expenses also grow at 7% after retirement too, how should we calculate the step- how long the corpus would last?

    • krishna kiriti said

      Can you please explain the first step as how to calculate on excel? I have the solution from FPSB, but I am not able to understand the process followed.

      • krishna kiriti said

        I am sorry for the trouble.I have found the formula used. Can you please explain how will I able to recognise that I need to use the annuity formula in that step.?
        thanks in advance.

  43. Manish said

    Dear vinay,

    1.)it is assumed that expenses will grow in pre as well as in the post retirement at the same rate throughout the period.

    2.)real rate of return has nothing to do with rental income.you just need to adjust inflation by taking the rate of return which u r getting on investment and inflation rate.Remember real rate of return is the concept at which your purchasing power increases while negative real rate indicates wealth destruction.

    3.)if expenses also grow at 7%,then the real rate of return comes to 1.87% and the no.of years the corpus would last are to be calculated as follow:-

    rate=1.87%
    pmt=1407106
    pv= -10601411
    mode= 1
    nper=?8 yrs. you can find the same solution in sample paper.I suggest you to solve some old papers first which you can find in the retirement segment and read all the comments while solving problem sums.

    • vinay said

      thank you manish sir,
      i am solving the old papers too. Coming to the problem – one last doubt (pls bear with me)- what is the assumption about the rent here? will it stop growing after retirement period or will it be growing at 7% after retirement?
      how to solve -1) if rent stops growing after retirement.
      2) if rent grows after retirement too.
      i am a little bit confused about this growing/not-growing rent after retirement period and its treatment. My logic is this growing/not growing rent affects the fund to be withdrawn every year and it should affect the corpus.
      please clarify.

  44. Manish said

    Dear vinay,

    1.)i think the rent stops growing after retirement while expenses continue to grow at 7% p.a, the same calculation is presented in the last step.

    2.)if rent grows after retirement then unlike fixed rent your corpus would last more as it helps you to cover major part of your basic expenses in post retirement period and no need to solve the same in this question.

    • vinay said

      manish sir,
      1) i thought the same- assumption is rent stops growing in the answer provided.
      2) yes, it may cover major part of the expense, but the point here is what is the method of calculation? – how to solve when rent grows?

  45. Manish said

    Dear vinay,

    in that case there has to be some formula or any other strategy to solve the same i can’t recall that now, meanwhile you have to wait for prashant sir’s response.

    • Dear all,

      Please find the solution

      Expense at retirement
      PV = 7,50,000
      N = 15
      I/Y = 7
      FV = 2,069,274

      House Rent at retirement
      PV = 2,40,000
      N = 15
      I/Y = 7
      FV = 6,62,168
      Hence, net requirement at retirement = 20,69,274 – 6,62,168 = 14,07,106

      Accumulation at retirement using FV of growing annuity = 1,06,01,411

      Number of years the accumulated corpus would last
      PV = 1,06,01,411
      PMT(BGN) = 14,07,106
      I/Y = RR
      N =? (Ans is 8 years)

      Post retirement expense will grow @7% and the same is with rent. Hence net amount will also grow at 7% rate. calculation for the same is as follows:
      EXP year 1: 2069274
      RENT year 1: 662168
      NET year 1: 1407106

      EXP year 2:2214123.18
      RENT year 2: 708519.76
      NET year 2: 1505603.42

      Same value can be calculated as 1407106*1.07. And that is the base if these calculations.

      Feel free to ask further.

      Regards,
      Prashant V Shah.

      • Manish said

        Dear prashant sir,

        Thanks for the clarity and working, especially for the doubt related to growing rent in post retirement period.

      • vinay said

        Prashanat sir,

        thank you verymuch for the explanation. Main point of confusion was the real rate of return in the last step. This real rate of return factors in the rate of growth of funds to be drawn from the corpus. am i right?
        Now my other problems 1) what if the rent grows at different rate than the expense, say 8% ? 2) what if the rent stops growing after retirement ?
        please clarify.

      • Dear Vinay,

        Your question is absolutely valid. To solve the same you will have to find net cash flow for individual year (preferably in excel or open office) and find the period in which it exhausts.

        Prashant.

      • vinay said

        Prashant sir
        thanks a lot. I clearly understood the problem and intricacies of the calculation with your and manish sir’s inputs. Thank you verymuch.

  46. vinay said

    Dear all
    I cleared (grade- C) the RPEB module today. Thanks a lot for the valuable inputs and questions posted here. Special thanks to Prashant sir and Manish sir for helping me with the problems.

    Exam was really tough and i could not attend all the problems. Much more practice, speed and understanding is required to solve the problems in given time.

    • aarvi1948 said

      Congratulations Vinay on your clearing RPEB module. In fact it was tough for me as well. Never the less you have cleared due to your hard work. With best wishes.,
      R Varadarajan

  47. Manish said

    Dear prashant sir,

    Request you to please help me with this question as follow:-

    Lynn received a rights offering to purchase one share of stock for Rs.60 plus 4 rights.since each right has a value of Rs.2,the total value of the stock under the rights offering is Rs.68.The existing stock has a cost basis of Rs.25.If the right is exercised,the cost basis for the new share is.

    a.)Rs.22
    b.)Rs.63
    c.)Rs.25
    d.)Rs.60

  48. Sridevi said

    Dear Prashant Sir, Manish Sir

    I appeared for Retirement module yesterday but couldn’t clear it. The exam was very tough and as said by other members the questions were really confusing and time consuming. I couldn’t attend all the questions. Can you People guide me how to clear this module. I’m really stuck with it. Also can you upload some questions from exam point of view i.e questions which are twisted. I’m fine with the theory part. Please suggest me some books which has got numericals on annuities.

    Regards
    Sridevi

  49. Sunit said

    please help in solving question no 15 19 and 20

    thanks in advance

    • vinay said

      Hi sunit
      Q.15- 10000 down payment. Company wants 10% interest compounded half yearly for the rest 190000.
      FC- cmpd- Set- begin, n = 8, i% = 5, pv = -190000. Solve for fv, fv = 280716.53.
      Calculate for 5 half yearly instalments with fv = 280716.53. set- begin, n = 5, pv = 0, solve for payment, emi = 48383.44.

      • udaycfp said

        Dear sir,

        why in solve you have taken i= 5 ?????????????????? why change in N both eq.

        Q.15- 10000 down payment. Company wants 10% interest compounded half yearly for the rest 190000.
        FC- cmpd- Set- begin, n = 8, i% = 5, pv = -190000. Solve for fv, fv = 280716.53.
        Calculate for 5 half yearly instalments with fv = 280716.53. set- begin, n = 5, pv = 0, solve for payment, emi = 48383
        Please ans asap because my exam in next week

        Regards
        uday

  50. Dear all,

    Can anyone help me with this sum

    A person age is 28, started investing 5000 pm in equity. After 5 years started additional investment of 5000 pm in debt with continuation of equity investment. At the age of 40 years, he transfers 50% accumulation of equity to debt. He increases investment in debt by 10000 pm. At the age of 60 he purchased deffered annuity plan which pays on monthly basis. you found that the rate of deffered annuity in order to cover post retirement expenses to be ______, if his current expenses are 25000 pm and returns on equity is 9.5%, debt 7.5% and inflation to be 5.5%, through out the period

    Regards

    • moneyacres said

      His liFe expectancy is 80 years

    • jasbir singh said

      Life expectancy is 80 years

      • Manish said

        Dear jasbir,

        with the life expectency of 80 yrs the problem may be solved as follow:-

        age = 28
        retire = 60
        a.phase = 32
        investments:-
        equity=5000 p.m
        debt = 5000 p.m

        ACCUMULATION OF EQUITY AFTER 12 YRS
        PMT= 5000
        RATE=9%/12
        NPER=12*12
        MODE=1
        fv =? 1308314.

        ACCUMULATION OF DEBT AFTER 7 YRS (FROM AGE 33 TO 39)
        PMT=5000
        RATE=7%/12
        NPER=7*12
        MODE=1
        FV=?548421.

        TRANSFER 50% ACCUMULATION OF EQUITY TO DEBT.

        ACCUMULATION OF EQUITY = 1308314
        LESS:-50% TO DEBT = 654157
        BALANCE = 654157

        ACCUMULATION OF DEBT = 548421
        ADD:-TRANFER FRM EQUITY =654157
        TOTAL =1202579

        ACCUMULATION OF EQUITY (FROM AGE 40 TO 59=20 YRS) COMES TO 7429700

        ACCUMULATION OF DEBT (FROM AGE 40 TO 59=20 YRS) COMES TO 10513716.TOTAL ACCUMULATION=17943417

        DEFERRED ANNUITY RATE:-
        NPER=20*12
        PMT=138681
        PV= -17943417
        RATE=0.58%

        CONVERT THE SAME INTO ANNUAL EFFECTIVE RATE =7.18%

      • jasbir singh said

        Dear Manish,

        Thanks for your valuable answer, but the condition is he increases his investments in debt by 10,000 Rs after 40 years of age as according to me pmt should 15000pm (5000+10000). Your calculations, according me are accurate. But I still seek recommendations from Prashant sir as there are more sums of dynamic asset allocations in retirement planning mostly for accumulation phase

        kindly Prashant Sir help us on this

        Regards

  51. moneyacres said

    His liFe expectancy is 80 years

  52. yash pandya said

    Dear all,

    Help with this sum,

    senior citizen have 2 fixed annuities for life time 220000 pm and 82000 pm, he also have 2 house he can let out second property at 25000 pm increasing by 8% pa. Now their expenses per month is 500,000 rs post retirement fix. he plan to take reverse mortgage loan on house where he currently live and its worth is 4 crore Rs. Bank have accepted to give 60% of loan @11.5%. he want to leave an estate of 2 crores after death. life expectancy is 95 years. inflation is 7.5 %. investment can fetch 9.5%. target are ______ and difference by _____

    options were achivable, and unachivable and the amount of difference

    • jasbir singh said

      Dear Yash,

      According to given condition he is in shortfall of almost more than 2 crore rs

      We can find out his current valuation by discounting all future income and expenses with investment rate.

      now for income payments, there are to fix annuity of 220000 and 82000 pm for next 35years who’s present value happens to be Rs 384,13,294.66 and for rent income 83,23,722.08, giving the net of Rs 4,67,37,016.74 @9.5% discounting rate

      now present value for future expenses of Rs 500000 and the estate of rs 2 crore are Rs 635,98,169.97 and Rs 8,34,710.40 respectively, net Rs 6,44,32,880.37

      now we can see there is short fall .almost 1.7 crore Rs which is not possible to cover for reverse mortgage loan as it can only fetch 27,20,908.23 and the maximum lumpsum he can be given is just Rs 15,00,000

      Thus it will be good for him to sell house which worth Rs 4 crore now and shift to second house in order to fill shortfall

      may be I couldn’t understand the question but let wait for others to reply on same

      Regards

  53. Dear prashant sir,

    can pl tel me what is rate to be consider for ppf 8.7 or 8.8 and how to cal maturity amt for ppf and what is d loan criteria

    What the Maturity date and value of a PPF accounted started on 31st March 2013 with Rs.1.00 Lakh and annual investment of Rs.1.00 Lakh on the same date every year till maturity – pl help me with ans

    • Dear zalak,

      Applicable rate for PPF is 8.8% for our exams and if PPF is started on 31st march 2013 then then maturity date will be 31st march 2028 with 16 payments as it requires 15 completed financial years and the same question is important for exam

      regards

      Jasbir singh

      • Ans for your question is 3244700.93,
        PMT=100000(END mode)
        N=16
        r= 8.8%
        fv=?

        question may arise that why 16, as it requires 15 completed years and to go with 15 completed years we did 1 extra payments and that too at the end of year. If it is done 1-april-2013 then we would just take 15 payments and that too in the beginning mode

        first loan can be taken from 3rd year till 6th year, that too of 25% of amount of the balance of preceding two years

        Now first withdrawal can be done from the 6th year

      • one more thing, interest on PPF is calculated on the lowest balance of end 5th day and last day of month,thus at the time of calculating FV of investments which are done on monthly basis we have to look whether the investment is done before 5th date of month or after

  54. pl rep for above question

    • Thanks Jasbir,
      but in one thing i am confused while cal ppf fv when we hv to use end mode or begin mode and 15 yr or 16 yr,
      say if first payment made on 1 july 2013 of ppf then here we use here 15 yr or 16 yr while cal

      • Dear zalak

        Good question to ask, remember this, if payment is done before or on 5th date of any month then the interest will be calculated on the payment for that month. If it is after 5th, interest will not be calculated. Now in this scenario investment is done on 1st july-13, the 15 financial years would be 1-april,14 to 31st-march,29. Seeing this I can say that last payment will be on 1-july,2028, thus we did 16 payments. we have to go through with this calculations

        PMT= 100000 (Bgn mode as month of july will be included)
        n= 16, rate = 8.8%
        fv= ? (35,30,234.611) Note:- this is the amount at 30-June,29 we need the amount of march-29, we have to remove interest of 3 months

        int factor of 3 months will be (1.007053)^3= 1.021309

        1.021309= 35,30,234
        1 = ?

        thus, ans will be 34,56,578.382

        In the exam you will also find the option of 3530234 but that’s not correct.

  55. DearJasbir Singh,
    pl let me know how u cal 1.007053 in above question .

  56. Sridevi said

    Dear Prashant Sir, Manish Sir

    I have cleared the Retirement planning module. I got 3 questions on asset allocation, two 4 marks questions on PPF, and nearly 7 more questions were from this blog. This blog and people around here have helped me so much in clearing this module. Thank You all. I have Tax planning module left. Pls suggest me some books for that also.

    Thanks
    Regards
    Sridevi

    • Dear sridevi,

      can you help us which questions were asked as I m planning to give retirement planning soon

      regards

    • Dear sridevi,
      pl let me know d question that come from this blog and about asset allocation question and ppf quet.as iam giving RPEB on 3rd oct

      • Sridevi said

        Hi Jasbir Singh and Zalakthakker

        I couldn’t recall all the questions. I’m giving some of the questions. I got same type of questions with different values from the model paper which is given in the fpsb website. the questions are Section 2 – Question Number 7, Ques 9, Section 3 – Ques 4, Ques 6, Ques 7, Ques 8, Ques 9, Section 4 – Ques 7.

        I got 4 questions on gratuity, two questions were to find the amt of gratuity payable(2 marks) and 1 question on taxable amt of gratuity whose is covered under the act and another one not covered under the act (3 marks).

        Other questions are

        3. Mr. A has gross annual salary of 9 lakh of which he saves 30% which include statutory EPF deduction, PPF and monthly systematic investment in long term MF scheme. Another 30% goes toward servicing of household & car loan & taxes. His financial planner advice him to accumulate 6 months household expense in liquid fund. HE change job and expect immediate rise of 20% in his gross income .You estimate that other heads would not change materially except his household expense which would rise by 5% due child education. How many months will it take to accumulate liquid reserve?
        a. 25 months
        b. 11 months
        c. 14 months
        d. 13 months

        4. An individual start investing immediately for 10 year annually Rs 80000 in the ratio 70:30 in equity and debt products. He expects the return from equity and debt to be 12.5% pa & 9.5% pa. during this period. To protect the wealth he rebalance the portfolio in 40:60 of equity and debt after 10 yrs and invest in same ratio annually rs 1.5 lakh for next 10 years. The return expected from equity and debt in this period subsides to 10.5% pa and 7.5% pa respectively. What could be his total investment at the end of the entire tenure of his investment?
        a. 60.38 lakh
        b. 60.31 lakh
        c. 70.42 lakh
        d. 61.58 lakh

        5..Find the yield of a 30 year annuity that gives 4% cashflow in the first year and an increase of 5% over the previous year for subsequent years”

        Solutions for this questions is given in this comments section. Comment no 22.

        PPF question.

        6. If 1 lac is deposited every year on the last working day of the FY i.e 31st March and the first deposit is made on 31st March 2013 when is the amount withdrawable.

        Solution in comments section. Comment number 25.

        The other PPF question i dont remember but both the questions came under 4 marks.

        7. A person age is 28, started investing 5000 pm in equity. After 5 years started additional investment of 5000 pm in debt with continuation of equity investment. At the age of 40 years, he transfers 50% accumulation of equity to debt. He increases investment in debt by 10000 pm. At the age of 60 he purchased deffered annuity plan which pays on monthly basis. you found that the rate of deffered annuity in order to cover post retirement expenses to be ______, if his current expenses are 25000 pm and returns on equity is 9.5%, debt 7.5% and inflation to be 5.5%, through out the period

        Jasbir singh has given this question in the comment 50 of this section.

        8.“Mr S aged 30 years is saving for the last 11 years in a Savings account giving a 3.5% p a. He is in the habit of increasing the amount by a fixed sum of Rs.2000 every year. If he deposits Rs.25000 in 12th Year what will be the corpus he will accumulate at the age of 60, if the rate of inflation is 11.68%
        Answers a ) 2980330 ( b ) 3251251 ( c ) 2523252 ( d )3322531
        Correct answer ( a ) 2980330″

        Solution for this is given by Vinayak Sir in the practice questions 3 in comment number 16.

        9. A&B both , both at 30 have current exp of 35000 pm and are saving Rs 5000 per month till their retirement at 60, with investment return of 8.5%. The post retirement inflation and and returns are 6.5% and 7.5%. If A decides to prepone his retirement by 5 years, how soon his corpus would exhaust compared to B.

        Dont remember the solution for this.

        These were the questions i could remember. The main problem with this paper is time constraint. You will not be able to finish the full questions paper bcos even in 2 marks there were so many numericals. So first try to attend all the questions which you are familiar with. Please thorough all questions given in the fpsb website. It will help you a lot.

        All the Best to both of you for the exams

        Thanks
        Regards
        Sridevi

      • Sridevi said

        Hi Jasbir singh and Zalakthakker

        One more question from this blog is

        1. Mr. A is of 35 yrs with spouse and a kid of an age 5 yrs. His strategic asset allocation is 50:35:15 in equity, debt and liquid. He is able to invest rs 1.5lakh pa immediately to work various life goal. At age 40 the allocation would change to 40:50:10 in equity, debt and liquid asset with annual investment going up to 2.5 lakh for 5 more years. At age 45, for next 10 year he adapts the conservative wealth protection allocation 25:70:5 in eq, debt & liquid asset with 3 lakh pa investments. The per annum return expected in this stage are; from equity : 12%.11% & 10%, from debt : 9%,8% & 7%, from liquid asset : 6,5%,5.5% & 4.5%.What amount could he accumulate by his age 55 years?
        a. 113.9 lakhs
        b. 97.21 lakhs
        c. 66.65 lakhs
        d. 117.91 lakhs

        Solution is given in Practice question for Investment planning.

        Regards
        Sridevi

      • akash singhal said

        sridevi,
        how do we solve question no.3 that u have posted (the first question i.e) ?

  57. Dear sridevi/Jasbir singh

    sol for question no 4 as post by u above pl check my ans for d same is corect,

    Future value of equity instrument with 70%
    N-10, I – 12.5 , PMT – 80000*70% , SOLVE FV – (1132650)
    Future value of debt inst. with 30%
    N-10, I – 9.5 , PMT – 80000%30% , SOLVE FV – (408924)
    TOTAL PORFOLIO – 1541574
    AFTER REBALANCING IN 40;60

    N-10, I – 10.5, PV -1541574*40% , PMT – 150000*40% , SOLVE FV – ( 2755902) ABT EQUITY

    N-10, I -7.5 , PV – 1541574*60% , PMT – 150000*40% , SOLVE FV – ( 3275070) DEBT

    TOTAL OF 2755902+3275070
    ANS – 6030972

    as option is given 60.31 pl tell me this ans is correct.

    one more qustion to Jasbir as u had given ip pl help me with above ip question post by sridevi mr A is 35 years spouse and kid of age 5…….,
    pl exppline me in detail with steps as in practice saction of ip only ans is given.

    In ppf question post by sridevi we hv to take 16 payment and end mode as u told ealier and mat date is 31st march 2028 and first withdrawal ( after 1st apr 2018) mat value is – 3244700.93 as post by u ( jasbir) pl tell this ans r correct

    if any one know d solution for question no 9 as post by sridevi A&B both have current exp of 35000………
    pl help me with sol

    • Sridevi said

      Hi Zalakthakker

      the answer for question 4 is correct and the answer for ip question is given in the comments section of practice questions for ip and here is the link for that. the solution is given by Manish in comment number 12.

      Important questions for Investment Planning – CFP

      Thanks
      Regards
      Sridevi

      • Hi Sridevi,/ prashant sir,
        As per sol post by manish ans is 113.9 but according to comment given below manish by neeraj jain ans is 117 so pl tell me d corect ans is 113 or 117 if i an doing rebancing in 3 rd step also i got d ans 113 ie

        after doing 2 rebalancing we get d 3189170 as portfolio amt,
        in 3rd time we have to take 3189170 in pv in 25;70;30 or not . and what is d correct ans for d same

    • Dear zalak,

      For question on PPF one prashant sir have already answered that it requires 5 completed financial year and for maturity value and date, it is correct.

      Now for question posted by sridevi mr A is 35 years spouse and kid of age 5, I would request prashant sir to clear the question of changing asset allocation of accumulated amount or keep it same

      now for question number 9,

      I have solved in this way

      for A, accumulated amount will be

      pmt 5000
      rate 8.5( monthly nominal rate)
      n 25*12(bgn)
      fv $4,934,679.04

      for B, accumulated amount will be

      5000
      8.5% (monthly nominal rate)
      30*12
      $7,791,750.37

      Now for expense, for A 35000*(1.065^25) and for B 35000*(1.065^30)

      for the years of A,
      pmt=168969.468
      rate= 0.077913% (monthly nominal real rate)
      pv= -$4,934,679.04
      NPER= ? (29.5301 years)
      thus 55+29.5301=84.53018091 (age at which money will exhaust)

      for B

      PMT= 231502.8157
      rate= 0.077913% (monthly nominal real rate)
      PV=$7,791,750.37

      Nper= ? (33.65725963)

      Thus 60+33.65725=93.65725963 (age at which money will exhaust)

      • My solution for question 9 may be wrong. m not that much confident. I would request Prashant sir, Manish and R Waradarajan or any other to help on this

      • Manish said

        Dear jasbir,

        Your methodology is correct but the nper which you have computed is in months not in years.

        1.)AGE AT WHICH CORPUS OF MR. A EXHAUST :-

        a.)nper=29 months (2 yrs and 5 months)

        b.)age at which corpus exhaust = 55+2 yrs and 5 months =57 yrs and 5 months

        2.)AGE AT WHICH CORPUS OF MR. B EXHAUST:-

        a.)nper=34 months (2 yrs and 10 months).

        b.)age at which corpus exhaust = 60+2 yrs and 10 months = 62 yrs and 10 months.

        3.)SO THE CORPUS OF MR A EXHUAST IN 5 YRS AND 5 MONTHS SOONER THAN B.

        In the question only post retirement inflation is given so we have to assume post-retirement inflation as pre-retirement inflation in order to inflate the current expenses of RS.35000 p.m.

      • Dear manish,

        Thanks… Now I got the Exact idea and the mistake I did… Thanks

  58. Thanks Jasbir Singh/ Sridevi
    for giving me quick rep.in quest no 8 post by Sridevi pl help me with detail sol as i had seen comment no 16 for sol but in that only final ans (2980330)is given.

    • Sridevi said

      You can use the following formula for calculating FV in case of a fixed amount increasing annuity:
      FV = A * Sn + D * ( Sn-N )/ I
      where A = Initial amount , D is the fixed increase, N is number of instalments, I is the rate and Sn is the FV of an annuity of Rs. 1 for N instalments and at I rate.
      For the above problem, we first calculate Sn. For this,N=41, I= 3.5%, PV=0, PMT = -1, END mode, we then get FV = 88.5095, which we take as Sn. D = 2000, A=3000 ( Initial amount). We then calculate as follows:

      FV = 3000*88.5095 + 2000 * ( 88.5095-41)/ 0.035 = 2980357

      This solution was given by Vinayak Sir. You can check in the comments section.

  59. HI,
    Today i passed RPEB with C grade. 3 mark and 4 mark qust was very tough i am not able able to solve that qust especially 3 annuity qust and 2 rev morgt quet was there .

    1 ppf question as post by me and sol given by jasbir was there . 5 qust from fpsb sample paper and question no 9 post by sridevi.

    one 3 mark qust
    For an investment product guaranteeing a fixed cash flow of Rs. 4 lakh per annum for 20 years beginning four years from the date of investment, what price should be fixed if the same can be invested in financial instruments which can yield 8.25 % p.a. for the four years and 7.5% p.a. for the remaining period of the product?
    Solution:
    Step 1: BGN, N=20, I= 7.5%, Pmt = 400000, PV= Solve (43,83,631), P/y= 1, C/y= 1
    Step 2: BGN, N=4, I= 8.25%, PV= Solve (31,92,437),FV= 43,83,631, P/y= 1, C/y= 1

  60. krishna kiriti said

    In 2013 Sample paper Section 2 – question 9 can somebody please explain how he as calculated the “Corpus to be accumulated on retirement” ?

    If you have no access to the question paper, the question is as below.Can you please provide a solution for the same.

    A 40 year old person spending Rs. 3 lakh p.a. plans to retire at age 63 and expects to live till 75 years. The basic inflation at 7% p.a. and lifestyle inflation at 1.75%
    p.a. are expected in the pre-retirement period. He starts investing for retirement at Rs. 30,000 p.a. in a 10% p.a. return instrument with immediate effect, and
    increases the contribution by 20% every year of the prior year investment amount. If the expenses post-retirement are curtailed by 20%, what maximum inflation
    would sustain his corpus till he survives, if the corpus is invested at 7% p.a.?

  61. HI KRISHNA,

    Solution: Step 1:- To calculate Future value of today’s expenses at retirement age 63. BGN, N= 23 (63-40), I =8.75 (7%+1.75%), PV= -300,000*80% (As, in post retirement period expenses will be 20% less) ,Fv= solve (16,52,282), P/y=1, C/y=1

    Step 2: Finding Future Value of Ongoing Investments till retirement age 63. As the investment of Rs. 30,000 p.a which is getting invested from now onwards (BGN Mode) is increasing every year by @ 20% p.a. So, in order to find out what amount will get accumulated till retirement age 63? , we have to use formula of Future value of Growing Annuity
    .
    Future value of growing annuity (FVGA) formulae for BGN mode calculation {Amount (1st Year Saving) * [{(1+R)^n – (1+G)^n} / (R – G)]} * (1+R) R – Required rate of return – 10% G – Growth rate in salary – 20% n- Term of Investment – 23 years – 1st year investment will be today at age 40 (BGN Mode) and last investment will be at age 62 (BGN Mode). So the total investment term will be 23 years
    .
    30,000 * [{(1.10) ^(23) – (1.20)^(23)} / (0.10 – 0.20)] =Rs. 1,71,87, 921 * (1.10) = Rs. 1,89,06, 713

    The amount to get accumulated by retirement age 63 is Rs.1,89,06, 713which will be utilized during the post retirement period.

    Step 3:- As per question language “what maximum inflation would sustain his corpus till he survives, if the corpus is invested at 7% p.a.?”For this we will first calculate inflation adjusted rate of return (i.e use step 2 of retirement planning calculation) and from the stated ANSWER we will calculate Inflation rate

    . Part 1: BGN, N=12 (75-63), I= Solve (0.876560), Pmt= -16,52,282, Pv= 1,89,06, 713 P/y=1, C/y=1 The “I” (0.876560%) indicates inflation adjusted rate of return.

    Part2: Inflation Adjusted rate of return = (1+ Return) / (1+ Inflation rate) Inflation Adjusted rate of return = 0.876560 % = 1.00876560 Return during post retirement period = 7% = 1.07 Inflation Rate = To solve (1.00876560) = 1.07 / (1+ Inflation rate) Inflation = 6.07%

  62. Akshaya Priya said

    Please let me know is there any changes made in cfp cm i have made the registration in the year 2011 and not able to clear modules since three times i had given the exam.

  63. kiriti said

    2013 sample paper
    Section 4 : Question 7
    A person at age 57 has accumulated Rs. 50 lakh towards retirement funds and opts for premature retirement. He purchases an immediate annuity for a total term of
    20 years, a fixed monthly amount for the initial period of 10 years and a provision to double the monthly amount in the second 10-year period. If the minimum yield
    guaranteed in the annuity is 8% p.a., what monthly amount he is expected to receive in the subsequent 10-year period?

    The solution as per fpsb for calculating the 2nd annuity was
    PV of the amount today for second 10-year annuity = PV((1+8%)^(1/12)-1,10*12,-200,0,1)/(1+8%)^10

    my doubt is why did they divide that (1+8%)^10 in the end.

    Thanks in advance.

    • R Varadarajan said

      Hallo Kiriti,

      In the First Part you have arrived at the PV ( today ) for the annuity of Rs.100 pm for the First 10 years (Rs.8397)
      In the second Part You have to arrive at the PV ( today ) of the annuity of Rs.200 pm. Just calulating Pv would give you the PV of the annuity of Rs.200 for 10 years at the beginning of the second phase of 10 years ( at the end of first phase of 10 years). To bring it down to the Present value as on today, you have to divide it by ( 1+0.08)^10. Trust this clarifies.

  64. Sweety Sharma said

    Please give me the solution of question no.15 ASAP

  65. abhinav said

    I have given rpeb module 2 times n till now i hav not cleared it….Prashant sir can you please suggest some good books on rpeb module..i need some additional questions for practise with proper solutions…

  66. abhinav said

    1.Suppose a person opened a ppf account during 2011-2012 and deposited 100000 at the end of every year,intrest rate is 8.8 %..At what date the ppf account will expire and what be the total amount during expiry of the ppf account..?

    2.Suppose a person opened a ppf account during 2011-2012 and deposited 100000 at the beginning of every year,intrest rate is 8.8 %..At what date the ppf account will expire and what be the total amount during expiry of the ppf account..?

    • Jasbir singh said

      Ans 1) as mentioned in question, 100,000 is deposited at the end of every year, that 1st payment is on 31-3-2012, accumulation will be Rs 32,44,701 on 31-3-2027( expiry date)

      Ans2)100,000@ bgn, that is 1-4-2011– accumulation will be 31,44,701 on 31-3-2026 (expiry)

  67. abhinav said

    These two questions are very important..i want the exact answers of these two questions..

  68. hi prashant sir

    pls provide me solution of these two questions in raip module..i m facing problem in these questions.
    I will be thankful to you.

    1..a client’s 20 year money back policy of sum assured rs 2 lakh has annual premium of rs 13672, Policy pays back 20% of s.a after each of first three 5-years survival periods and another 40% of s.a on surviving full term. The client has received the third money back. You estimate the gross returns presently in the policy considering reversionary bonus of rs 50 per thousands s.a. you compare the cost benifit if the client pays all premiums and survives the policy and also gets rs 150 per thousand s.a as loyalty bonus. you conclude that _________.
    a)the overall return improves marginally by 1.15% p.a
    b)the additional in flow on 5 future premiums would amount to over 19% p.a returns
    c) the additional in flow on 5 future premiums would amount to nearly 30% p.a returns
    d) the additional in flow on 5 future premiums less opportunity cost would amount to nearly 12% p.a

    2..an industrialist started a project on 1st nov 2009 with own capital of rs 1crore.He arranged a project of rs 1.5crore by a back on 1st july 2009 at a rate of 12%p.a. the terms of finance were quarterly invested.only payments up to six quarter and the repayment of premium in three equal installments from the end of seven quarter along with interest on the loan outstanding.he infuced fresh own funds towards working capital of rs 20 lakh on 7th december 2009 and rs 30 lakh on 4th november 2010.the project completed with a profit of rs 4.5crore on 6th september 2012.find the return generated by the project…
    a)22.59%p.a.
    b)30.16%p.a.
    c)30.57%p.a.
    d)32.37%p.a.

    regards
    manisha

  69. Dhanu said

    Dear sir
    I am not able to understand how the real rate of return came to 0.92% in question number 42..please find the question and the solution below.appreciate your earlier reply

    Q)A 45-year old man spends Rs. 7.5 lakh p.a., almost the amount he earns, to maintain his family. He expects his expenses to rise by 7% p.a. He has not saved for retirement. He has a second house which he wants to rent at Rs. 20,000 p.m. immediately, the rent expected to increase by 7 % p.a. You advise him to create a corpus by his age of 60 by investing the rent received in an instrument yielding 9% p.a. at the end of every year. You estimate the number of years the accumulated corpus would last taking the received rents post-retirement into account.

    How long your corpus would last?

    pmt=1716959
    real rate=0.92%
    pv= -10601411
    nper=?6 yrs.

  70. dhanu said

    Dear Sir

    Please find below the question and the options

    Thanking you

    A 45-year old man spends Rs. 7.5 lakh p.a., almost the amount he earns, to maintain his family. He expects his expenses to rise by 7% p.a. He has not saved for retirement. He has a second house which he wants to rent at Rs. 20,000 p.m. immediately, the rent expected to increase by 7 % p.a. You advise him to create a corpus by his age of 60 by investing the rent received in an instrument yielding 9% p.a. at the end of every year. You estimate the number of years the accumulated corpus would last taking the received rents post-retirement into account. The same is ____.

    (a)over 4 years
    (b)over 7-1/2 years
    (c)8 years
    (d)over 3 years

  71. akash singhal said

    HELLO SIR,
    IN QUESTION 8, WHY HAVE U NOT TAKEN NOMINAL RATE ?

  72. akash singhal said

    please help me solve this question:

    An executive purchased an annuity for a lump sum Rs. 85 lakh when he was of 53 years and had in dependents a non-working spouse of age 48 and a son of age 25. On reaching age 60, he expects at least one, himself or his spouse, to survive till 85 years and contracts an immediate life annuity with return of purchase price at Rs. 10.15 lakh p.a. vested against the purchase price of Rs. 1.61 crore. What return is expected from the vesting date?
    a. 6.73%
    b. 5.76%
    c. 4.25%
    d. 5.17%

    • vinay said

      Hi Akash
      annuity per year- 1015000, purchase price- 16100000, Return of purchase price(fv)- 16100000, duration- 30 years
      set= begin, n= 30, pv= -16100000, pmt= 1015000, fv= 16100000, i%= solve, answer= 6.728, option-a.

  73. akash singhal said

    maximum exemption for gratuity is currently 10 lacs right?

  74. akash singhal said

    PRASHANT SIR,
    PENSION IS ASSUMED TO BE RECEIVED IN THE BEGINNING OR N THE ENDIF THE QUESTION IS SILENT ON THIS?

  75. akash singhal said

    If the income from a senior citizen savings scheme for a 63 year individual is Rs105000/-and there is no other income then the taxable amount is:

    1)Nil

    2)5000

    3)10000

    4)500

    Answer is 5000 or NIL?

  76. Sangeetha Ravada said

    Nil is the answer
    Income Slabs Tax Rates
    i. Where the total income does not exceed Rs. 2,50,000/-. NIL
    ii. Where the total income exceeds Rs. 2,50,000/- but does not exceed Rs. 5,00,000/- 10% of the amount by which the total income exceeds Rs. 2,50,000/-.
    iii. Where the total income exceeds Rs. 5,00,000/- but does not exceed Rs. 10,00,000/- Rs. 25,000/- + 20% of the amount by which the total income exceeds Rs. 5,00,000/-.
    iv. Where the total income exceeds Rs. 10,00,000/- Rs. 125,000/- + 30% of the amount by which the total income exceeds Rs. 10,00,000/-.
    Surcharge : 10% of the Income Tax, where total taxable income is more than Rs. 1 crore. (Marginal Relief in Surcharge, if applicab

  77. akash singhal said

    Please help me out with this question-

    Seema aged 35 years requires 20lakhs at age 55. If she starts today using an annualstepped up method. Calculate how much she needs to save in first year to ensure that her investment plan corresponds with salary growth. Assume interest rate 8%, growth 10%and inflation 4%:

    1)17596

    2)17922

    3)17900

    4)18900

    • R Varadarajan said

      Dear Akash,

      Assuming the investment in the first year is Re1, the Future value at the age of 55 would be Rs.111.59331 as per growing annuity method
      FV = {1* (1.10^20-1.08^20)/(1.10-1.08)}*1.08 = 103.32714*1.08=111.593311

      Investment required for a FV Rs.20,00,000 = 2000000/111.593311 =17922.22
      Answer 2.

  78. akash singhal said

    Q.) In current times of recession, Sujit’s company is going through financial crisis.Due to continuous losses the management has decided to offer Voluntary retirement to itsemployees on 1-4-2009. The company has got the VRS scheme approved through ITAuthorities. He has asked for your advice on net tax proceeds out of his VRS amount of Rs. 20 lakh. The same will be—-

    Rs. 13.21 lakh b.

    Rs. 17.60 lakh

    Rs 14.90 lakh

    Rs. 15.50 lakh

    the answer is 14.90 lakhs but how?

  79. akash singhal said

    Ajay and Bela Mahera have two children ages 5 and 7. The Mehera’s want to start saving for their children’s education. Each child will spend 6 years at college and will begin at age 18. College currently costs Rs. 20000 per year and is expected to increase at 6% per year. Assuming the Maheras can earn an annual compound return of 12% and inflation is 4%, how much must the Mehera’s deposit at the end of each year to pay for their children’s educational requirements until the youngest is out of college? Assume that educational expenses are withdrawn at the beginning of each year and that the last deposit will be made at the beginning of the last year of the younger child’s college education.

    JUST GIVE THE FINAL ANSWER OF THIS QUESTION.

  80. akash singhal said

    If the inflation rate is 4.9% and tax rate is 30% the required rate of return to maintain the value of an investment will be ___________.
    Choose one answer.
    a. 8%
    b. 9%
    c. 7%
    d. 10%

    Please help me with this question.

    • R Varadarajan said

      Answer is c. 7%

      Post tax return = 0.7 ( i e 1-0.3) should match the Inflation (4.9% )to maintain the value of investment

      Return required = 4.9/0.7= 7 %

      • can you explain this?

      • R Varadarajan said

        Tax Rate is 30% and inflation rate is 4.9%. To maintain the value of investment means, there should not be any erosion in the value of the investment due to inflation. The net return from the investment is the post tax return and this should cover the inflation so that the value of investment remains unaffected. The post Tax Return is 70%, of the actual return, which should match the inflation 4.9%.. If 70% is 4.9% 100% would be 7% as worked out. You can recheck the same like this – If actual return is 7% , Post Tax Return would be 7%*0.70 = 4.9% which is the inflation rate

        Trust this clarifies your doubt

  81. akash singhal said

    thank you.

  82. dhanu said

    Dear sir

    pls help me out with this question

    Thanx n regards

    senior citizen have 2 fixed annuities for life time 220000 pm and 82000 pm, he also have 2 house he can let out second property at 25000 pm increasing by 8% pa. Now their expenses per month is 500,000 rs post retirement fix. he plan to take reverse mortgage loan on house where he currently live and its worth is 4 crore Rs. Bank have accepted to give 60% of loan @11.5%. he want to leave an estate of 2 crores after death. life expectancy is 95 years. inflation is 7.5 %. investment can fetch 9.5%. target are ______ and difference by _____

  83. Harshil said

    Dear Sir,
    Are these questions and answers as per the new format of Begin mode.

  84. Rajat said

    Q4. An industrialist started a project on 1st nov 2009 with own capital of rs 1crore. He arranged a project of rs 1.5crore by a back on 1st july 2009 at a rate of 12%p.a. the terms of finance were quarterly invested.only payments up to six quarter and the repayment of premium in three equal installments from the end of seven quarter along with interest on the loan outstanding.he infuced fresh own funds towards working capital of rs 20 lakh on 7th december 2009 and rs 30 lakh on 4th november 2010.the project completed with a profit of rs 4.5crore on 6th september 2012.find the return generated by the project…
    a)22.59%p.a.
    b)30.16%p.a.
    c)30.57%p.a.
    d)32.37%p.a
    Can any one give me the solution of this que. very important

  85. abc said

    1.Suppose a person opened a ppf account during 2011-2012 and deposited 100000 at the end of every year,intrest rate is 8.8 %..At what date the ppf account will expire and what be the total amount during expiry of the ppf account..?

    Can any one provide me the solution of this questions.

    • Nihar said

      The account matures on 1st April 2027
      Maturity amount is Rs.2890350

      • aarvi1948 said

        Although PPF is a 15 years scheme, there will be 16 deposits until maturity. As suggested by you, the maturity date is 1st April 2027. But there will be 16 deposits starting from 2011/12 at the end of the year, till 2026/27 and with the interest rate at 8.8% the Maturity Value would be Rs.3244700. ( use CMPD – n = 16, I=8.8% PMT = -100000 ) FV will be 3244700 ( or Alternatively n =15, I=8.8% Pv=-100000, pmt= -100000 FV = 3244700 )

  86. abc said

    1. Mr. A invests Rs.50,000 pa in debt mutual fund which gives 8.5% return pa. Investment will increase at 10% pa. Mr. B is investing Rs.40,000 in the same debt mutual fund along with investing Rs.10,000 in growth oriented equity fund which has return of 11% pa with same growth rate in investment and for the same investment period. The difference between the effective return earned is

  87. sonali said

    Can Anyone have solution for this Que.

    Q.There was a question in asset allocation. A person has 15 years to retirement. First 5 years he wants to allocate 2 lacs, then next 5 years 3 lacs and then 5 lacs for last 5 years in equity and debt in following ratios: first 5 years 70:30 for equity and debt, next 30:70 and again back to 70:30. The rate of interest for first 5 and last 5 years for equity is 19 and 7 and middle order block of five years is 11 and 9 for equity and debt. Calculate the retirement corpus.

  88. sonali said

    Q Mr. A invests Rs.50,000 pa in debt mutual fund which gives 8.5% return pa. Investment will increase at 10% pa. Mr. B is investing Rs.40,000 in the same debt mutual fund along with investing Rs.10,000 in growth oriented equity fund which has return of 11% pa with same growth rate in investment and for the same investment period. The difference between the effective return earned is

    Q.A& B both aged 30 plan to save Rs.10000 pm for 30 years till their retirement. A starts immediately, invests for 10 years, stops investment and allows the investment to grow. But B starts 10 year and continues till retirement. Who gets more and how much.

    If anyone know how to solve this Questions please help me with same as earliest.
    thanks

    • Nihar said

      Hi Sonali.

      Use CMPD function & GCF to solve q1. Post which xirr function in excel to arrive at rate of return. The diff is .51% (B being higher than A)

  89. ASHISH PRATIHAR said

    Thanks for Sample Paper and solutions. From Last one week i am following this website. I glad to inform you that today i have successfully clear my RPEB exam with B grade. Many question are there from mock paper in website.

    Once again thanks 🙂

    With Regards,
    ASHISH PRATIHAR

  90. hello to everyone out here..can anybody who has already passed risk and insurance module share in a few questions which you attempted in the exam…prashant sir,could you please provide the answers to practice questions in insurance module or if you have any additional questions please share.Thanks

  91. Darshan said

    Can anyone post the questions if anyone has appeared for the retirement exam.

  92. Darshan said

    Prashant Sir,

    Can you tell me which is the good book to study retirement planning. I have FPA book.

    • Darshan said

      Sir Plz reply the book to refer for retirement planning.

      • rahulmj08 said

        HI darshan i have used the study materials of FPA Learning for ” Risk Analysis & Insurance Planning” & “Retirement Planning & Employees Benefits” – . I found them extremely good, precise and these surely helped me to clear the exam in single attempt.GO for it and all the best for the exam.

      • Darshan said

        Thanks Rahul for the info. Can u share some questions of retirement planning if u remember any it will be a great help from u.

  93. Darshan said

    Has anyone given retirement planning exam recently? Can anyone share some questions

  94. rmanohar11 said

    Dear Friends,

    How can I get study materials of FPA Learning for ” Risk Analysis & Insurance Planning” & “Retirement Planning & Employees Benefits”, what is source eg:- Online or from any book store.

    Can you guide me please I am going to appear in next week.

    Regards,

    Manohar.

  95. Dear Prashant Sir,

    Please me for the below mentioned sums to solve and understand with proper steps:

    Q1. Mr. A wants to purchase a car costing 8,50,000/- , viewing her budget she determines she can afford to pay 15000/- per month for 3 years towards the car, ROI is 1% per month for 3 years.
    How much she can afford to borrow?
    Select nearest option.

    A. 4,50,000/-
    B. 5,50,000/-
    C. 8,00,000/-
    D. 6,00,000/-

    Q2. Mr.A have two mortgages loan option before, his Interest and other conditions are the same for both except that one has a repayment term of 15 years and term of 30 years. He wants to evaluate the EMIs for both terms. All other conditions being the same repaying the loan in 15 years instead of 30 years requires EMI i.e (options)

    A. half the size o 30 years loan payment.
    B. less the twice as large as 30 years loan payment.
    C. more than twice as larges as 30 years loan payment.
    D. twice as large as 30 years loan payments.

    Q3. Mr.R wants to purchase a car costing 8,00,000/- with down payment of 15% balance to be refinanced by a car finance company. Loan is for 45 monthly installments. Loan is @ 13% on monthly reducing balance.

    A. What will be his EMI?
    B How much interest will he pay in entire loan?
    C. If there is foreclosure of 3% penalty , What he will pay to car company after 35 installments.

    Q4. Mr. V purchases house for 25 lakhs, 5 lakhs down payment, balanced to be repaid in 15 years on monthly installments @ 9.5%
    House was purchased on 1st April 2012, he made 1st installment on 1st May 2012 regularly paying installments.

    A. what is his EMI?
    B. Interest paid on entire loan?
    C. OPENING balance on 01.04.2015 and Closing Balance as on 31.03.2016? and How much benefit he can claim under SEC 80C and 24(b) for AY 2016-2017?
    D. O/B as on 01.04.2016 and C/B as on 31.03.2017, how much benefit he can claim under SEC 80C and 24(b)
    for FY 2016-2017?
    E. If he wants to prepay the loan for purchase of a house in DEC 2015 and a prepay close penalty of 2%how much she will pay as on 1.12.2016?

    Q5. Mr D makes a 2,00,000/- payment of repayment @ 3rd year of loan and maintains the same EMI, than for how many years her loan repayment will get reduced?

  96. KOMAL GUPTA said

    hi please give solution for question no 2.. also has anyone given retirement exam recently? i need the updated question bank with solutions

  97. Vansh said

    Plz give me d solutions of d above questions

  98. Vansh said

    I need solutions of d above questions

  99. NANDINEE VAIDYA said

    PLEASE HELP ME ON EXAMPLE 6

  100. Rohit Vishwakarma said

    Sir,
    Could you please help with sum no 18
    Ms. Reshma is 35 years old and plans to retire at 50. Her life expectancy is 60 years. Ms. Zarina her Financial Planner, estimates that her client will require Rs.65000 in the first month after retirement. Inflation rate is 3% p.a. and the rate of return is 5% p.a. What will be the savings per year required in order to meet this?
    a) Rs. 328300
    b) Rs. 345897
    c) Rs. 315894
    d) Rs. 320458

Leave a reply to Prashant Shah Cancel reply